Dispelling myths about global warming

CO2 did not drive the rapid warming of the 20th century.

Story submitted by Stan Robertson

The difference between a good idea and a bad idea is often a quantitative matter. For example, many people would think it a good idea to replace internal combustion engines with electric motors. But if the intent is to reduce the burning of fossil fuels then switching to electric motors would not help unless the electricity was generated without burning fossil fuels. Some people think that it has been a good idea to use corn to produce ethanol for a fuel, however, I am not one of them because the energy return on investment is either negative, or minuscule at best.Ā  From the standpoint of greenhouse gas emissions, it is a horrendous loser. It may be a biofuel and cleaner burning, might help ameliorate ozone problems and etc, but considering that nearly a gallon of oil is consumed in addition to the gallon of ethanol produced and burned, it is a quantitative loser. (Not that I care at all about the CO2.)

One of the ideas that seems to be widely believed is that human produced greenhouse gases, chiefly CO2, has dominated the warming of the earth in the last century. It is a simple quantitative matter to show that this is completely false.

According to the calculations of the UN IPCC, a doubling of the atmospheric concentration of CO2Ā  (with an accompanying rise of other greenhouse gases) would reduce the outgoing infrared radiation from the earth by a net 2.7 watt/m^2 at the top of the atmosphere. This is known as the ā€œclimate forcingā€ that will occur along with a doubling of the CO2. This is a relatively straightforward, but messy calculation. I have repeated the IPCC calculation for CO2 and obtained a larger number, but after including the IPCC adjustments for other greenhouse gases and the effects of sulfate aerosols accompanying coal burning, we agree. It is important to note that the surface temperature increase that will accompany the CO2 is proportional to the logarithm of the CO2 concentration. Thus while CO2 concentration is increasing exponentially with time, the temperature only increases linearly.

In order to maintain equilibrium with the incoming UV/VIS radiation received by the earth, the surface temperature would need to increase enough to allow it to radiate an additional 2.7 watt/m^2 at the top of the atmosphere after any CO2 doubling. At a nominal surface temperature of 15 CĀ  (288 K), the earth surface radiates about 390 watt/m^2 on average, but the radiation that exits the top of the atmosphere is only 240 watt/m^2. Thus the earth would need to produce an additional (390/240)x2.7 watt/m^2 = 4.4 watt/m^2 at the surface in order to offset the direct effect of doubling the atmospheric CO2. At 288 K, the earth radiates an additional 5.4 watt/m^2 per 1CĀ  of temperature rise. Thus the direct effect temperature increase of a CO2 doubling would be 4.4/5.4=0.8 C.

At the present 0.5% per year rate of increase of CO2 it will take about 140 years to double its concentration. But as we all know, a 0.8 C temperature increase in 140 years is not the result that the UN IPCC is alarmed about. The IPCC climate models include large positive feedback effects that raise their expected temperature increase into the range 2 ā€“ 4.5 C, with their most probable value at about 3 C.

There are four main arguments against this: (1) We have already had half of a 2.7 watt/m^2 climate forcing since pre-industrial times. That has been accompanied by only 0.8 C temperature increase.Ā  As shown below, there are reasons for believing this to be due primarily to natural causes. (2) There is no evidence that confirms the existence of any large feedback effects since the end of the last deglaciation. (3) The rate of temperature increase within the past century has been within the bounds of normal climate variability and (4) as shown below, the heating effect of CO2 has been quantitatively inadequate to explain the actual warming that has occurred in the last century.

There have been two periods of rapid warming that account for most of the warming that occurred in the last century, as shown below.

Letā€™s examine the first of these rapid warming periods first. By 1944, the atmospheric CO2 concentration had increased from the pre-industrial level of about 280 ppm up to 310 ppm. At that time the concentration was increasing at a rate that would require about 600 years to double. The fraction of a doubling climate forcing that would have occurred by 1944 would have been log(310/280)/log(2)=0.15 and this would have contributed at a rate of 0.15×2.7 watt/m^2 per 60 decades, or 0.0068 watt/m^2 per decade. Itā€™s direct warming effect at the surface would thus be only (390/240)x(0.0068 watt/m^2 per decade)= 0.01 watt/m^2 per decade. This would have raised the temperature by (0.01 watt/m^2 per decade) /( 5.4 watt/m^2 /C) = 0.002 C per decade. This is such a pitifully small fraction of the 0.174 C per decade rate of heating that occurred 1917-1944 that it is pretty clear that CO2 had nothing to do with the warming of the first half of the last century.Ā  Even the IPCC climate modelers concede this point.

But there is still more to be learned from that period. Apparently some natural phenomenon allowed the earth to absorb energy at a significant rate and produce the temperature increase of the first half of the century. Letā€™s see how much that might have been. To begin, the earth would have had to take in enough heat to at least produce the additional surface radiation that would accompany a temperature rise of 0.174 C per decade 1917-1944. This would be (5.4 watt/m^2/C)x(0.174C/decade) = 0.94 watt/m^2 per decade. This is already 94X the CO2Ā  heating rate.

But, in addition, as shown by both the ARGO buoy system and heat transfer calculations, at least 700 meters of upper ocean can respond to heating on a time scale of a decade. The additional amount of heat required to raise its temperature by 0.174 C per decade would be c*d*0.174C, where c= 4.3×106 joule/m^3/C is the heat capacity of sea water andĀ  d= 700 m, or 5.2×10^8 joule/m^2. Dividing by the number of seconds in 10 years, this would be an average of 1.7 watt/m^2 per decade. But since it would start at zero, it would have to end at 3.4 watt/m^2 per decade in order to attain this average. This should be added to the 0.94 watt/m^2 per decade surface radiation losses by the end of the warming period. So the total heating rate would have to ramp up by 4.3 watt/m^2 per decade to provide the warming that actually occurred in either of the rapid warming periods.Ā  This is 430 times the direct CO2 surface heating for 1917-1944.

Since essentially the same rate of temperature increase occurred 1976-2000, we can compare 4.3 watt/m^2 with the heating that might have been caused by CO2Ā  in the last part of the last century. From 1944 to 2000, the CO2 concentration increased from 310 ppm to 370 ppm, with a doubling time of about 140 years. The corresponding climate forcing that would have caused, at the surface, would be (390/240)x(log(370/310)/log(2))x(2.7 watt/m^2)/14 decades = 0.08 watt/m^2 per decade.

Due to the higher rate of growth of CO2 concentration in the second half of the 20th century, this is 8X as large as the direct surface heating effect caused by CO2 in the first half. Nevertheless, it is still some 54 times smaller than the rate of heating that actually occurred.

These straightforward calculations make it painfully obvious that CO2 forcing is not what drove the two periods of rapid heating during the last century. Until there is some understanding of the natural causes of these rapid warming periods and their inclusion in the climate models, there is no reason to believe the models.Ā  This is simple first year physics.

===========================================================

Stan Robertson, Ph.D, P.E, retired in 2004 after teaching physics at Southwestern Oklahoma State University for 14 years. In addition to teaching at three other universities over the years, he has maintained a consulting engineering practice for 30 years.

0 0 votes
Article Rating
139 Comments
Oldest
Newest Most Voted
Inline Feedbacks
View all comments
grumpyoldmanuk
March 25, 2013 4:44 am

A quick google search reveals Stan Robertson the Architecture professor and graphics designer and Stan Robertson the Scots folk-singer. niether is likely to be this Stan Robertson. Can we have a thumb-nail bio?

Ed_B
March 25, 2013 4:46 am

“In order to maintain equilibrium with the incoming UV/VIS radiation received by the earth, the surface temperature would need to increase enough to allow it to radiate an additional 2.7 watt/m^2 at the top of the atmosphere after any CO2 doubling.
I stopped reading right here. WTF?
After 5 years of reading here I guess I still don’t get it. All that I see is that the earth needs to change its vertical and horizontal heat distribution system a bit. That would mean a tiny bit wider belt of thunderhead clouds, with a slight increase in frequency, a slight increase in ocean surface heat towards the poles, with a tiny increase in temperature at the poles.
I still think we are looney if we think our measurement systems are accurate enough to track those tiny changes. For example, the net net overall average temperature change might only be 0.3 C or so. How do you measure that when 40 C swings are common annually, and in some cases, daily?
I’m sorry, but I just walked out of this ‘professors’ classroom muttering “another idiot”.

March 25, 2013 4:53 am

Soot melts the snow even when the temps are below freezing. Sun hits the soot, the soot retains the heat and the snow beneath it melts. It’s the soot, not the CO2 that we need to clean-up.
Plants need CO2 and we need plants.

Peter Miller
March 25, 2013 5:02 am

This, of course, once again confirms the existence climate cycles – the great heresy of the Global Warming Cult.
The cult’s great ‘truth’ is positive feedbacks, despite the fact there is little or no evidence of them and none at all in the geological record.
Something interesting may be happening with global temperatures right now, as according to UAH, they are falling in an almost unprecedented fashion.
http://discover.itsc.uah.edu/amsutemps/

Jimbo
March 25, 2013 5:07 am

It may be a biofuel and cleaner burning, might help ameliorate ozone problems and etc,…

Ameliorate ozone problems? Now that could be a problem.

Now, a new study in Nature Climate Change warns that biofuels using fast-growing trees (polar, willow, and eucalytpus) could also exacerbate ground-level ozone pollution.
http://news.mongabay.com/2013/0109-hance-biofuel-ozone.html

Abstract
Impacts of biofuel cultivation on mortality and crop yields
Ground-level ozone is a priority air pollutant, causing ~ 22,000 excess deaths per year in Europe1, significant reductions in crop yields2 and loss of biodiversity3. It is produced in the troposphere through photochemical reactions involving oxides of nitrogen (NOx) and volatile organic compounds (VOCs). The biosphere is the main source of VOCs, with an estimated 1,150ā€‰TgCā€‰yrāˆ’1 (~ 90% of total VOC emissions) released from vegetation globally4. Isoprene (2-methyl-1,3-butadiene) is the most significant biogenic VOC in terms of mass (around 500ā€‰TgCā€‰yrāˆ’1) and chemical reactivity4 and plays an important role in the mediation of ground-level ozone concentrations5. Concerns about climate change and energy security are driving an aggressive expansion of bioenergy crop production and many of these plant species emit more isoprene than the traditional crops they are replacing…………….
http://www.nature.com/nclimate/journal/vaop/ncurrent/full/nclimate1788.html

lurker passing through, laughing
March 25, 2013 5:07 am

In the pre-AGW era thinking, it was believed the temperature history showed century scale movements much larger than we have experienced in the last ~ two centuries. It would be interesting to find out how that record and general agreement were discarded. My bet is that they were simply erased, a la hockey stick. But it would be worth documenting never the less.

StephenP
March 25, 2013 5:13 am

I see that we are all doomed, according to the outgoing chief government scientific advisor, Sir John Beddington:
http://www.telegraph.co.uk/earth/environment/9951872/World-faces-decades-of-climate-chaos-outgoing-chief-scientific-adviser-warns.html
However all is not lost if his replacement Sir Mark Walport comes up with what he says:
“He is also a champion of open access to scientific research, the idea that the fruits of public or charity-funded research should be available to everyone, free of charge. He recently implemented a policy at the Wellcome Trust to penalise researchers who do not make their work available free of charge.”
Walport said he was “delighted and honoured” to have been appointed. “Science, engineering and technology have transformed the infrastructure of the modern world, and have a vital role to play at the heart of policy making,” he said.
“They are critical both to economic recovery and growth, and to addressing many of the greatest challenges of our time, such as environmental change and the ageing population. I look forward to working with colleagues both inside and outside government to ensure that the best possible advice can be provided from the most expert sources, based on the strongest evidence, to facilitate the wisest possible policy decisions.”

March 25, 2013 5:14 am

But there is still more to be learned from that period. Apparently some natural phenomenon allowed the earth to absorb energy at a significant rate and produce the temperature increase of the first half of the century.
Yes, natural phenomenon , balance of the warm and cold water flowing in an out of the Arctic Ocean, moving polar jet stream back and forth.
http://www.vukcevic.talktalk.net/NAP-SST.htm
Why would balance change ? Geology .
Geology? Sun, actually.
http://www.vukcevic.talktalk.net/SSN-NAP.htm
but nobody knows how and why, so CO2 as a handy scapegoat.

Jimbo
March 25, 2013 5:15 am

At the present 0.5% per year rate of increase of CO2 it will take about 140 years to double its concentration. But as we all know, a 0.8 C temperature increase in 140 years is not the result that the UN IPCC is alarmed about. The IPCC climate models include large positive feedback effects that raise their expected temperature increase into the range 2 ā€“ 4.5 C, with their most probable value at about 3 C.

I’m still not alarmed.

Abstract
Temperatures in tropical regions are estimated to have increased by 3Ā° to 5Ā°C, compared with Late Paleocene values, during the Paleocene-Eocene Thermal Maximum (PETM, 56.3 million years ago) event. We investigated the tropical forest response to this rapid warming by evaluating the palynological record of three stratigraphic sections in eastern Colombia and western Venezuela. We observed a rapid and distinct increase in plant diversity and origination rates, with a set of new taxa, mostly angiosperms, added to the existing stock of low-diversity Paleocene flora. There is no evidence for enhanced aridity in the northern Neotropics. The tropical rainforest was able to persist under elevated temperatures and high levels of atmospheric carbon dioxide, in contrast to speculations that tropical ecosystems were severely compromised by heat stress.
http://www.sciencemag.org/content/330/6006/957

Alan D McIntire
March 25, 2013 5:19 am

Another point- that 390 watts is assuming that the earth acts as a blackbody. If it did, radiation out would quickly adjust to equal radiation absorbed- if that were the case, nighttime temperatures would quickly drop to the 2.7 K we get from “big bang” background radiation, and daytime temps would rise to 350 K, killing us all. If climate modelers knew anything, they would quickly throw in Newton’s law of cooling,
http://www.ugrad.math.ubc.ca/coursedoc/math100/notes/diffeqs/cool.html
and adjust the coefficient k to match current rates of cooling at night and warming during the day. Playing around with a simplified model, I discovered that starting with everything in balance, and a sine wave increase in temps during the day and decrease at night, then increasing the incoming radiation by a set amount- you gradually get a new balance of course- under the new balance, nighttime temps increase more than daytime temps- it’s obvious this must be the case, else we’d reach that 2.7K ” big bang” nighttime temperature.

JA
March 25, 2013 5:19 am

The planet in the past has experienced far more extreme temperature paradigms (both much warmer and much colder) over far, far, longer periods of time than we are witnessing today.
Also, the earth has experienced CO2 levels in the past far higher And far lower than we have today and NO ONE seems to know what caused that. (Did dinosaurs drive SUVs and heat their dens with power generated by coal fired plants?)
In each instance the planet did not burn up and turn into Venus, nor did it turn into a giant iceball.
And NO ONE can explain what caused these historical climatic periods, NO ONE knows what caused them to reverse course – as they ALL did .
So, pray tell, why is any HONEST person concerned in the slightest about today’s climate?
If science still has much to learn about the multitude of variables, and their interaction, that affect climate (as evidenced by their ignorance in explaining the historical climate) , how can they presume to predict the future climate?
After all, if you cannot explain the historical climate, one cannot honestly claim they have the knowledge to predict the future climate.
By the way, is not water vapor the MOST IMPORTANT greenhouse gas, given that it constitutes well over 90% of all the greenhouse gases?

Greg Holmes
March 25, 2013 5:20 am

A woderfully concise explanation, the thing about physics is that it has no political colour, it just is.

Martin Audley
March 25, 2013 5:25 am

Although I understand you are talking hypothetically, about the maths, the phrase:
“Thus while CO2 concentration is increasing exponentially with time, the temperature only increases linearly.”
would be better written:
“Thus if CO2 concentration were to increase exponentially with time, the temperature would only increase linearly.”,
or better still, to match closer to the real world:
“Thus if CO2 concentration were to increase linearly with time, the temperature would only increase by the log of that increase.”,

March 25, 2013 5:30 am

It remains true to this day: there is no evidence that increasing the concentration of CO2 in the atmosphere, particularly in the presence of convection, increases the Earth’s surface temperature. I like the way the young Russian Physics PhD candidate put it to me on the plane to Penang. “There is no mechanism,” she said. The idea that added agents of cooling will throttle or restrict or trap or block outgoing radiation is akin to an intelligence test.

Solomon Green
March 25, 2013 5:44 am

This morning on the BBC Today programme Sir John Beddington, who has just retired as the UK Government’s Chief Scientific Adviser, said that current climate reflected the global CO2 levels of 25 years ago and that today’s increased CO2 levels would be reflected in global temperatures 25 years from now.
This implies that global warming lags rising CO2 levels by 25 years. I do not remember seeing this before in any climate change models. Have I missed something or is this a new hypothesis that has been specifically designed to explain the apparent failure of global temperatures to increase over the last 14 to 16 years?

johnmarshall
March 25, 2013 5:45 am

You take no account of the PROPORTION of atmospheric CO2 is down to us which is 3-4%, the rest is from natural producers from Volcanoes to insects through plants at night etc., etc., etc..
There are no peer reviewed papers proving CO2 drives climate/temperature but plenty concluding no connection between CO2 and temperature. None of the GHE theory predictions have been found or happened so your insistance that CO2 drives climate/temperature, though you claim small, is built on sandy wind blown ground. The constant proof of the connection between CO2 and climate from the alarmists are model outputs all of which rely of factoring in CO2 as the driver. This is NOT proof but a circular argument and fails.
We live on a dynamic planet with a chaotic climate system driven by insolation NOT some trace gas vital for life.

March 25, 2013 5:48 am

Er . . . “Thus while CO2 concentration is increasing exponentially with time, the temperature only increases linearly.” ?????
CO2 is NOT increasing exponentially with time. Starting out totally wrong.

Mike M
March 25, 2013 6:10 am

And not only the issue comparing rates of warming in the modern record, the instances of COOLING in the presence of higher CO2 concentration for the modern and ancient record obliterates the alarmist position. The debate over the CO2 lag in the Vostok ice record always seems to focus on the periods of temperature rise, (with the alarmist guess saying that CO2 somehow ‘took over’ the forcing later..) – but never about the decline in temperature after CO2 had already risen.
If increased CO2 concentration actually provided any amount of forcing to ‘help’ temperature go up then, once that concentration was higher – why didn’t it seem to have any affect on preventing temperature from coming back down?
From a systems viewpoint, you could postulate the exact opposite that the ice record is showing us that increasing of CO2 dampens whatever was causing temperature to go up. In other words, CO2 is a lagging negative feedback and any delayed negative feedback is always going to produce an oscillation. Warming produces more CO2, the CO2 builds up to some level putting the brakes on temperature which then starts going down and CO2 lingers driving temperature even further down that whatever forced it up in the first place. Works for me…

John Peter
March 25, 2013 6:16 am

I wonder where Bob Tisdale is to explain the two separate increases in temperature 1917 to1944 and 1976 to 2000.

Richard M
March 25, 2013 6:19 am

I like the way the young Russian Physics PhD candidate put it to me on the plane to Penang. ā€œThere is no mechanism,ā€ she said. The idea that added agents of cooling will throttle or restrict or trap or block outgoing radiation is akin to an intelligence test.
This is the point I have been making for several years. CO2, and all GHGs, also have a cooling effect on heat in the atmosphere. While they do warm the surface slightly they also cool the atmosphere. These two complimentary effects balance out which is exactly what we’ve seen over a billion years of history.

Richard111
March 25, 2013 6:30 am

Solomon Green says: March 25, 2013 at 5:44 am
I heard the same and thought another one crying the sky is falling, not now, but soon. Sheesh.
I am also not happy with Stan Robertson’s story. Could we please have an explanation in simple first year physics on the ‘forcing’ of CO2 compared with the performance of H2O.

Rob Potter
March 25, 2013 6:42 am

This essay treats the global temperature as a function of radiative physics and – as far as I can tell – does a decent job of looking at the basic mechanism of the radiative energy balance, but this is not what drives global atmospheric temperatures and therefore I am not sure that this is the right premise on which to refute the IPCC claims.
Most of our near-earth atmosphere (where the measurements used to estimate the “global temperature” are made) has an energy balance dominated by evaporation and precipitation since the majority of the earth’s surface is covered either permanently or intermittently by water. I am not even sure the radiative physics equations can be applied above the surface layer either because of the impact of cloud formation on energy transfers as well as vertical air movements.
It is quite clear that the observed warming cannot – in any way – be considered to be unusual in the past 60 years, and thus cannot be ’caused’ by anthropogenic increases in atmospheric CO2. However, I don’t think we can use these radiative physics equations to refute the IPCC claims. To be honest, if it were that easy, I suspect we wouldn’t have had to go on for so long with this disaster of a theory!
Thanks for the essay, Stan, the numbers are really useful and i will keep them for future reference but I am sad to say that I doubt it will sway too many people.

H.R.
March 25, 2013 6:51 am

grumpyoldmanuk says:
March 25, 2013 at 4:44 am
“A quick google search reveals Stan Robertson the Architecture professor and graphics designer and Stan Robertson the Scots folk-singer. […]”
========================================================
Could be either. It doesn’t take a rocket scientist to see that CO2 doesn’t have what it takes to drive climate; that the assumed positive feedbacks used in the models were an incorrect assumption; that catastrophic anthropogenic CO2-based global warming isn’t happening.
Heck! I’m still waiting for Greenland to warm up past the good ol’ days of the Vikings so I can perhaps do a little homesteading over there. Greenland is almost there, but it’s still a little colder than Greenland’s farming heyday back when.
I’m thinking a Scots folk-singer might be a little overqualified to pick apart CAGW, but I have to agree with you here. A short bio tacked on the end, as in most other guest posts, is always nice.
Best regards from a slightly-grumpy-from-shoveling-snow-at-5:00am H.R.

rgbatduke
March 25, 2013 6:59 am

After 5 years of reading here I guess I still donā€™t get it. All that I see is that the earth needs to change its vertical and horizontal heat distribution system a bit. That would mean a tiny bit wider belt of thunderhead clouds, with a slight increase in frequency, a slight increase in ocean surface heat towards the poles, with a tiny increase in temperature at the poles.
All it needs to do? Do you think that the Earth is some sort of sentient being with intentions? What matters is what it actually does.
I can see lots of ways that the Earth could actually cool with increasing CO_2. I can imagine it remaining neutral. I can think up ways where it warms a little bit, or warms a lot. None of this ultimately matters — what matters is what it does, and once we understand it, why. At the moment we do not understand it — we understand pieces of it. The “changes in horizontal and vertical heat distribution” you so glibly refer to is one of the bases of the GHE — it is computable (subject to some assumptions, but they are assumptions like “assuming that the laws of quantum physics are valid”, and “assuming that the atmosphere doesn’t nonlinearly react to the change”). It is the all things being equal baseline assumption that small changes in CO_2 concentration don’t cause the entire system to suddenly reconfigure and that radiation physics per se holds.
Of course, even here there are assumptions that are not written down. The most important ones (as the article notes) are feedback with water vapor (nothing else really matters as far as feedback is concerned, as even if there is methane feedback, it breaks down quickly to CO_2 in the upper atmosphere and is biologically broken down to CO_2 in the ocean too fast to be a player itself), modulation due to OTHER atmospheric components, e.g. soot, dust, aerosols (many of which are net cooling), and although the article does NOT discuss it in any detail, modulation of albedo both from ice/snow and from clouds tops. There are also slower acting players in the game whose quantitative effects we cannot even guess at — orbital modulation, oceanic turnover, the effects of phase changes in the major global atmospheric oscillations.
Could these all combine to produce net cooling on top of weak CO_2 driven warming? Sure. Could they all combine to produce net warming on top of weak CO_2 driven warming? Absolutely. Could these all combine to produce net warming or net cooling on top of no change in CO_2? Obviously (glancing at the presumed global thermal record and pretending that the error bars are small enough that it has some reasonable probability of being valid) it has, repeatedly, in the past produced net warming and net cooling both on top of no significant change in CO_2 levels.
And ultimately, if you read past the beginning, this is all the article talks about. One could summarize the article as:
“Natural variability in the twentieth century produced a warming rate in the first half that almost precisely matched the warming rate in the second half, with a flat-to-cooling hiatus in between. In the first half of the century, CO_2 was completely irrelevant to the warming. In the second half, its direct warming effect was a larger but still very small fraction of the observed warming. In both cases, since we cannot actually compute the contribution of natural factors to the warming or explain the warming observed in the first half (or the cooling of the LIA, or the warming of the MWP, or the cooling before that, or the warming of the RWP, or the cooling from the mid-40’s through the mid-70’s in the twentieth century, or…) we cannot conclude that high-sensitivity CO_2 warming was responsible for the temperature rise in the second of the two rises in the twentieth century.”
At that, he pulled some of his punches. He did not point out, for example, that temperatures reached a 1950’s-like plateau as of the 1997-1998 super-ENSO event and have been more or less flat for some fifteen to sixteen years even as CO_2 levels have increased significantly. He didn’t point out that they were nearly flat — except for a few bobbles associated with e.g. Mount Pinatubo — for the decade preceding that, and that far from smoothly increasing in consonance with CO_2 concentrations, the apparent explanation is that the ENSO event is responsible for more or less 100% of the observed warming from this era in a punctuated equilibrium model.
In the end his conclusion is identical to yours — that CO_2 increases are not a good explanation for the observed temperature rise — so far. Of course this year temperatures could spike up 0.3 C and resume a “torrid” pace, or they could continue their doldrums, or they could even drop. A sudden rise would constitute evidence supporting the CAGW point of view. A sudden drop would constitute negative evidence. Continuing flat temperatures constitute weak negative evidence (accumulating to eventually become strong evidence).
So if we’re going to anthropomorphize the Earth and imagine that it is going to do what it “needs” to do according to your belief system (or theirs!), let’s all agree to try to let the Earth speak for itself instead of claiming to speak for it. Science works better that way.
rgb

RichieP
March 25, 2013 7:11 am

StephenP says:
March 25, 2013 at 5:13 am
‘However all is not lost if his replacement Sir Mark Walport comes up with what he says:
ā€œHe is also a champion of open access to scientific research, the idea that the fruits of public or charity-funded research should be available to everyone, free of charge. He recently implemented a policy at the Wellcome Trust to penalise researchers who do not make their work available free of charge.ā€ ‘
————————————————————-
I shouldn’t get too excited about Walport if I were you. He is a convinced warmist.
‘Sir Mark Walport, Director of the Wellcome Trust, responded to the series, saying: “Al Gore described climate change as ‘An Inconvenient Truth’, but the findings of this study offer a very convenient truth. Reducing greenhouse gas emissions is not only essential to help tackle climate change, it is also an important way to improve public health.
“We urge world leaders, when they meet in Copenhagen next month, to take account of the health impacts of different mitigation strategies, and to work towards a solution that improves both the health of our planet and its people.” ‘
http://www.wellcome.ac.uk/News/2009/News/WTX057672.htm
and (the source of the above link):
‘This is also the man who, in July 2011 was adding to his voice to the complaints that the BBC was giving “too much weight to fringe views on issues such as climate change”.
So, as we shiver through the unseasonable weather, hardly daring to venture out into the biting easterly wind, these fools prattle and prance with the quasi-religious mantras.
You will get nothing sensible from Walport. He is pure establishment. But then that is the joy of being a warmist ā€“ you can keep repeating the same old, same old, never having to admit you are wrong. And the idiots in government will always give you a nice cushy job.’
http://www.eureferendum.com/blogview.aspx?blogno=83748

Philip Mulholland
March 25, 2013 7:12 am

Careful Mike M, You’re in danger of starting the next Global Cooling scare.
You’ve just generated the perfect mechanism to explain the sudden onset of the next glaciation. /Sarc

MartinGAtkins
March 25, 2013 7:19 am

Joe G says:
March 25, 2013 at 4:53 am
Soot melts the snow even when the temps are below freezing. Sun hits the soot, the soot retains the heat and the snow beneath it melts. Itā€™s the soot, not the CO2 that we need to clean-up.

It depends on the angle of the sun relative to the sooty surface. At the poles the angle of the suns rays are very acute so although it would play a part in the melting it’s the long wave back radiation that does the persistent melting. Even on a cloudy day sooty snow will melt and patches of soot will drill their way down bellow the surface of the clean snow. Back radiation is a product of the atmosphere and so it is always directly overhead of the surface.

rgbatduke
March 25, 2013 7:22 am

CO2 is NOT increasing exponentially with time. Starting out totally wrong.
Ah, so which is it? Is it a vast international conspiracy to fake the direct observational evidence of a roughly exponential increase, or is it that the instrumentation — all of it at multiple observatories — is faulty? Would you have any actual source of evidence supporting this statement (since there are multiple sources of evidence to the contrary)?
Sheesh. And people wonder why “warmists” make fun of “deniers”.
I’ve got a good idea. Let’s agree not to issue a blanket statement that some measured result is false on WUWT without at the same time presenting precisely the same evidence that would be required to publish a paper refuting it. I’m not talking about criticizing speculative, complex theories, I’m talking about direct measurements.
For example, a good (if too short) list of things that should be out of bounds without evidence include direct instrumental measurements such as:
a) Satellite measurements of temperature. Criticizing this is fine — if you have evidence sufficient to write a paper that could in principle convince e.g. Roy Spencer that UAH has an error. Otherwise, this is a nearly direct instrumental observation and while it is not beyond any doubt it is beyond any reasonable doubt unless and until a specific problem is discovered.
Things that do not count as specific problems include “but it isn’t returning the result that I want it to return”, by the way.
b) Atmospheric CO_2 concentration. Again, this is a direct instrumental observation (actually the average of a whole series of many direct instrumental observations from many observatories). You can argue about the published numbers, sure, or can claim that there is surplus CO_2 on the top of volcanic mountains in Hawaii, but in the end, you need to both experimentally confirm this on site and present alternative measurements that suggest that it is, in fact, introducing a systematic error. Which is a fine subject for a published, peer reviewed paper.
c) Top of the atmosphere or bottom of the atmosphere spectra. This is again right in there with satellite measurements of temperature. The instrumentation is relatively straightforward, and it returns the results it returns. You may not LIKE the results they return, but if you don’t like the raw measurements, the onus is on you to demonstrate a systematic problem with the instrumentation, which is (IMHO) very, very unlikely to happen. But if you do, many journals would be happy to publish it so that your assertions can be openly debated.
d) SLR. This actually goes for the “warmist” side much more than the skeptical side. I’m sick and tired of hearing assertions of multimeter SLR by thus and such a time given actual instrumental records stretching back 140 years with a total SLR over that entire interval of nine whole inches, with a current rate of roughly 3 whole millimeters a year (just over an inch a decade), and with similar rates clearly observable in the historical record in almost precise correspondence with the temperature records illustrated in the top article above.
Note well that I’m perfectly happy to see things like “ice melt rates” derived from complicated gravitational and surface measurements criticized, and in fact these “instrumental” results HAVE HAD recent papers published criticizing them the “right” way. Instrumentally derived results can indeed be faulty, but they aren’t faulty because somebody says so, they are judged faulty because further evidence is accumulated that more or less confirms an error in what was done before.
Or did you mean that the rate isn’t exactly exponential? Like this matters?
rgb

March 25, 2013 7:28 am

Today I woke up to the BBC saying: “climate sceptics do not think CO2 causes warming” (or similar words).
I know we don’t think that, and worse, I’ve told everyone in the BBC who I could think to tell from the Chairman to the legal department.
And now … what do I find when I turn to WUWT … a clear demonstration that sceptics support the science of CO2 warming. We just reject the non-science of Hansen, Mann and the BBC.

March 25, 2013 7:35 am

According to the chart, the two peaks are ~60 years apart. AMDO/PDO cycle comes to mind. I think what we have witnessed is the belated non-recognition of this cycle, first described in 1996, just a few short years after the celebrated discovery of AGW, placement of the UNFCCC, and it’s brainchild, the IPCC. The IPCC is famous, and in fact chartered, to paper-over anything which does not comply with AGW, such as the discovery of AMDO/PDO a few years after it was chartered.
As I read Stan’s contribution, which was well done in math, that 60 year cyclicity stood out like a sore thumb. Grudgingly, looking at the lack of heating over the past ~15 or more years, some cheerleaders have actually now conceded that AGW heating is being masked by a natural cooling trend which might continue for a decade or two before resuming AGW heating.
Which might be partially correct, assuming that the sun really does have a miniscule effect on gaia climate. But we really don’t know what role the sun plays in the AMDO/PDO cyclicity yet, do we? And if the sun has gone all quiet on us now, and predictions are that it will go even quieter, something which we noticed centuries ago matching up with the LIA, then what tipping point might yet be in our 21st century at a half-precession old interglacial happening at a 400kyr eccentricity minimum?
Well the last one (MIS-11) went long, running somewhere between 22-32kyrs. If CO2 was involved in that extension (a) there is not much evidence to support it in the paleo record, and (b) if it was then there had to have been some natural occurrence, we may not have been on-stage just yet. So, it could happen. But MIS-19, ~800kyrs ago, didn’t go long. That would make it a coin toss unless CO2 is the heathen devil gas it is made out to be.
Which is what makes this so fascinating. If you do not want to in any way impede mother nature’s side of the coin from landing up, instead of down, you will want to remove any chance of impeding the onset of the next ~90kyr long glacial. You will then set about removing said climate security blanket. Which, from a human speciation perspective, might not be such a bad idea. However, if you leave the so-called climate security blanket in place, even grow it, then you increase the chances of the AGW side of the coin landing up, reducing the opportunity for a return to glaciation and concomitantly for genus Homo speciation. Honestly, as I look around me these days, I am not at all sure that would actually be a good thing.
Decisions……..Decisions…………

robbcab
March 25, 2013 7:39 am

What I find just as, if not more, interesting about the chart you used is the “pause” in warming between ~1944 -~1976.
Didn’t I read somewhere recently that a “standstill” of 30 years while CO2 emissions continue to rise invalidates the models?
Things that make you go Hmmmm…

robbcab
March 25, 2013 7:43 am

What I find just as, if not more interesting in the chart of warming rates is the “pause” in warming from ~1944-~1976. Didn’t I read recently that a “standstill” of 30 years while CO2 emissions continue to rise invalidate the models?
Things that make you go Hmmmm…

Steve Keohane
March 25, 2013 7:47 am

Mike M says:March 25, 2013 at 6:10 am
Good point Mike. I have always had a problem with temperatures crashing at the highest levels of CO2 in the ice core proxies, and then pretending that CO2 is driving the temperature, doesn’t work for me.
I could see a scenerio where a little CO2 enhances temperature, but at some point replacing WV with CO2, the latter a less efficient GH gas, causes cooling.

Jeff Alberts
March 25, 2013 7:47 am

It may be a biofuel and cleaner burning, might help ameliorate ozone problems and etc

Pedant note: Etc. means “and others”, so you’ve basically said “and and others”.
/pedant

Gail Combs
March 25, 2013 7:57 am

Ed_B says:
March 25, 2013 at 4:46 am
ā€œIn order to maintain equilibrium with the incoming UV/VIS radiation received by the earth….
I stopped reading right here. WTF?….
>>>>>>>>>>>>>>>>>>>>>>>>>>>>
He is taking THEIR assumptions and calculations and showing that even using that as a starting point the Climate Models flunk first year physics because the leave out the elephant in the room, aka what are the natural causes of climate variability that effected the first half of the 20th century.

March 25, 2013 8:02 am

Reblogged this on Rashid's Blog.

Joe
March 25, 2013 8:06 am

For all those who are “reading so far” then stopping on principle because the essay includes IPCC-think phrases, perhaps you should read through and think what’s being said.
Certainly, as I read it, the author is NOT endorsing the IPCC model. In fact, he’s doing exactly the opposite and using its own premise to invalidate itself. And any model which invalidates itself can’t be correct except by chance.

Mark Hladik
March 25, 2013 8:07 am

Well said, JA (5:19 AM, 25 March 2013).
Add to that the probable 13% atmospheric concentration of CO2 during the Rodinian glacial episodes (three documented) and one should be left with more questions than answers.
And for those “faint sunners” who continue to point to the difference in solar output, remember that your hypothesis hinges on heat being “trapped” by CO2, not on the amount of radiation being emitted at the source and absorbed by the receiver.
Plus, when the sun was even fainter, in the Early Proterozoic, with similar carbon dioxide concentrations, the average global temperatures were more like what we are accustomed to, not super-cold, and not super-hot.
There is absolutely no correlation between atmospheric carbon dioxide concentrations and average global temperatures.
Period.
Mark H.

Editor
March 25, 2013 8:12 am

Stan Robertson: The only thing I ask: when you use one of my graphs, please provide a link to the source of the graph:
https://bobtisdale.wordpress.com/2011/12/12/part-2-do-observations-and-climate-models-confirm-or-contradict-the-hypothesis-of-anthropogenic-global-warming/
Regards

jabre
March 25, 2013 8:20 am

” considering that nearly a gallon of oil is consumed in addition to the gallon of ethanol produced and burned”
Let me guess – this is from the infamous Patzek, Pimental Berkley paper.
I stopped reading at that point. One of the reasons I consider WUWT credible is that the posts generally are considerate of the references and do not choose junk science to substantiate a non-credible argument.
I end my reading with the sentence above and conclude that this post is junk.

Stephen Wilde
March 25, 2013 8:26 am

Ed_B said:
“After 5 years of reading here I guess I still donā€™t get it. All that I see is that the earth needs to change its vertical and horizontal heat distribution system a bit. That would mean a tiny bit wider belt of thunderhead clouds, with a slight increase in frequency, a slight increase in ocean surface heat towards the poles, with a tiny increase in temperature at the poles”
Agreed. Have been saying as much for years.

March 25, 2013 8:30 am

Does anyone else share my puzzlement about the way in which people write about the current, observed increase in CO2 concentration in the atmosphere. It seems to be mandatory to say “CO2 is increasing exponentially” with an implied “with time” – though this is seldom ever stated explicitly. In his third paragraph Stan Robertson states ” Thus while CO2 concentration is increasing exponentially with time, the temperature only increases linearly. ” . What always puzzles me is that if you actually plot CO2 concentration against Time it plots as a sloping monthly sawtooth – or possibly sinusoid – which has an almost exactly constant slope when measured over a number of years. At present it is increasing effectively linearly with time. In fact, if you explore Mauna Loa data using various regression models the underlying (annual) response is remarkably close to being an exact straight line. Higher order terms are virtually absent. Where, then, does the oft quoted “exponential” character of the atmospheric CO2 concentration arise?
Am I being particularly thick? My understanding of the meaning of “exponential” as a description of the behaviour of a curve is that it implies a continuously changing slope. If the exponential term is positive the slope increases continuously. One can of course also fit a regression model in which the underlying idea is that the measured quantity increases (or decreases) towards an asymptote at a rate that diminishes as the x variable increases, eventually approaching zero.
If CO2 is indeed increasing in concentration linearly with time then the underlying physics leads to the conclusion that the increase in temperature that this produces will tend towards an asymptote. This is what almost everyone seems to believe, and arguments abound about just how large or small the exponential constant is. It is usually expressed in the context of global climate as the increase in CO2 concentration that is required to produce a factor 2 increase in temperature rise.
The corollary is that if CO2 /is/ increasing exponentially the (short term) response of temperature increase should be linear.
]
Can someone please sort me out?
Robin

AlecM
March 25, 2013 8:31 am

Anyone who accepts that the Earth emits real radiative energy at the black body flux which assumes a sink at absolute zero, is a lunatic. This makes all climate alchemists lunatics. They believe that they’re right because pyrgeometers measure temperature then convert it by the S-B equation to Power, also the potential flux to a sink at absolute zero. This is also lunacy but includes the meteorologists as well!
The real operational radiative emissivity is ~0.16 (63 W/m^2/396 W/m^2). This is easily predictable because the GHG thermal emission is black body, a simple quirk of physics, and annihilates the same emission bands from the surface.
There is no GHG forcing except for non self-absorbed trace gases and water vapour side bands. Much more IR goes to space via the Atmospheric Window.
The modellers exaggerate GHG absorption by a factor of 6.85 = 157.5 W/m^2/23 W/m^2. None of it is CO2 – three separate bits of physics. This is the cause of the imaginary warming and imaginary feedback.
Don’t believe me? Go and talk to any process engineer or professional physicist who has done a bit of experimental heat transfer measurement in industrial plants.

Stephen Wilde
March 25, 2013 8:39 am

And I don’t see Ed_B’s comment as anthropomorphic as does rgbatduke.
All that happens is that the atmosphere obeys the Gas Laws as it must.

Asmilwho
March 25, 2013 8:42 am

There’s a Stan Robertson who posts here
“The Definitive Global Climate Change (aka Global Warming) Thread — General Discussion and Questions”
http://www.peakprosperity.com/comment/149210
Seems to be the same guy

March 25, 2013 8:45 am

Pffffft…. the GHE hypothesis is hogwash anyway… all of this is moot.

Alan D McIntire
March 25, 2013 8:54 am

rgbatduke says:
March 25, 2013 at 7:22 am
CO2 is NOT increasing exponentially with time. Starting out totally wrong.
Ah, so which is it? Is it a vast international conspiracy to fake the direct observational evidence of a roughly exponential increase, or is it that the instrumentation ā€” all of it at multiple observatories ā€” is faulty?
The amount of CO2 in the atmosphere has supposedly increased from 280 ppm around 1900 to 390 ppm now, about a century later. 390/280 = 1.393. If the rate of increase is exponential, the exponent is 0.00332. That’s a pretty small exponent, hardly distinguishable from linear over a small time frame like a century or two.

bobl
March 25, 2013 8:56 am

CO2 rising exponentially? It’s barely quadratic!

March 25, 2013 9:11 am

Exactly. All that fuss is based on 30-year long AMO positive phase. Exactly such one occurred in 1910-1945, when warming reached 0,7C. No model is able to simulate this, since models are not able to simulate natural variations like NAO, AMO or PDO cycles.

Lester Via
March 25, 2013 9:13 am

How does one arrive at the “relatively straightforward, but messy calculation” – doubling the CO2 will reduce the outgoing IR radiation at the TOA by a net 2.7 W/m2? Some spectroscopists seem to disagree, saying, based on actual measurements, that it is a far smaller figure. Does anyone have a link to a detailed explanation?

March 25, 2013 9:13 am

Martin Audley says:
March 25, 2013 at 5:25 am
As rgbatduke already said, the CO2 levels in the atmosphere are measured quite accurate. In my opinion even with the best and most rigourisely quality controlled network of the world for any known measurement. That shows that the CO2 levels are increasing significant, slightly exponential, over time since at least 1900.
One can discuss a lot of findings like the temperature record, where the quality control is often questionable or even absent. But CO2 measurements at individual baseline stations have an accuracy of 0.1 ppmv and between stations there is not more difference than +/- 2 ppmv for yearly averages. The main differences are because of a lag in the increase with altitude and latitude. The SH levels lag the NH with 1-2 years, which points to the NH as main source of the increase.
Here a comparison between temperature, human CO2 emissions and CO2 levels in the atmosphere over the period 1900-2006:
http://www.ferdinand-engelbeen.be/klimaat/klim_img/temp_emiss_increase.jpg

March 25, 2013 9:28 am

Trashing the CO2 hypothesis without looking further a green field is not moving the science forward, and eventually it is simply a waste of time.

March 25, 2013 9:29 am

Robin Edwards says:
March 25, 2013 at 8:30 am
Where, then, does the oft quoted ā€œexponentialā€ character of the atmospheric CO2 concentration arise?

Indeed hardly visible in the curve, thus the exponent is very small, as Alan D. McIntyre calculated. It is better visible in the average year by year increase:
http://www.ferdinand-engelbeen.be/klimaat/klim_img/dco2_em.jpg
From 0.8 ppmv/yr increase in 1960 to near 1.8 ppmv/yr in 2006 and 2.1 ppmv increase last year…
The rate of change of both human emissions and the increase in the atmosphere are increasing linearly which gives a slightly exponential increase of both the total emissions and the increase in the atmosphere. The remarkable effect is that increase in the atmosphere remains at a very stable ratio with the human emissions:
http://www.ferdinand-engelbeen.be/klimaat/klim_img/acc_co2_1900_2004.jpg

March 25, 2013 9:30 am

Until there is some understanding of the natural causes of these rapid warming periods and their inclusion in the climate models, there is no reason to believe the models.

Natural recovery from the Little Ice Age? Seems to me everyone is running around trying to find something on which to “blame” the recovery of the climate from one of the coldest periods in the Holocene.

john robertson
March 25, 2013 9:40 am

Seems like Team science is so bad, that even when one uses their standards as defined, to demonstrate the divergence of results, others get bogged down in the shadow science of the IPCC.
As I understand Stan Robertson, even if one uses the IPCC’s assumptions and mathematics, their argument is F.U.B.A.R by reality.
I appreciate rgbatduke’s point we do not know and cannot say using the data we currently have.
However I am fed up with the corruption, wilful blindness and fraud, that is climatology.
The false certainty and lust for power that runs thro this perversion of science, has me gnashing my teeth and looking for a scapegoat to rend.Not a great place for a so called science.
And a very uncomfortable place for a man who wants to believe in civil society.
How did we sink so low as to create a religion over plant food?

March 25, 2013 9:49 am

AlecM says March 25, 2013 at 8:31 am
Anyone who accepts that the Earth emits real radiative energy …

Can you explain the appearance of frost with, say, an air temperature of 35 deg F?
.

Ed_B
March 25, 2013 9:50 am

Stephen Wilde says:
March 25, 2013 at 8:39 am “And I donā€™t see Ed_Bā€™s comment as anthropomorphic as does rgbatduke.
All that happens is that the atmosphere obeys the Gas Laws as it must.”
Thanks. I agree. I was “feeling” the gas laws. Ha.. too long a mechanical engineer I guess.
Stephen, your descriptive models have helped me, along with Willis’s great analysis of the equatorial thunderhead storms doing vertical heat transport above most of the CO2, and also Bob Tisdales work on the earths natural horizontal ocean heat distributuion capability.
The earth is dynamic system, and I am stongly uncomfortable with the static approach describing anthroprogenic CO2 radiative forcing.

AlecM
March 25, 2013 10:22 am

_Jim: it’s all about the atmospheric window. Of net IR from the Earth’s surface, 2/3 rds goes to space. At night this still happens so you go sub sero with a clear sky.
Air temperature 15 Ā°C with all radiative heat transport requires a surface temperature of -13.5 Ā°C.
Hence the cold surface is covered with dew or frost and triggers fog.
There is no GHG blanket – it’s a pathetic bit of physics by people who haven’t a clue about real heat transfer.

nc
March 25, 2013 10:36 am

How come the difference between man caused C02 and natural C02 is hardly ever noted even on this site related to C02 and temperature levels. When the question is asked it seems to be ignored. For sake of argument lets go with the IPCC temperature rise of 3 degrees, C or F who cares. Lets say mans contribution to C02 level is 3%. Mans contribution to temperature rise is then .09 degree who cares.
I guess because there is virtually no difference except to someone’s bank account the .09 is conveniently ignored.
To me all the seemingly endless arguments, discussions, research, activists marches and so on just goes poof, if the above is enforced.
Please someone tell me what I am missing here?

Rob Ricket
March 25, 2013 11:09 am

It seems the is a sifi author by the name of Kim Stanley Robinson, who wrote a series of novels dealing with green issues, social justice and climate change. If this is our man, he’s had a major change of heart.
Just a shot in the dark.
http://www.kirkdorffer.com/ontheroadto2008/2007/05/kim-stanley-robinsons-science-in.shtml

March 25, 2013 11:09 am

vukcevic says:
March 25, 2013 at 9:28 am

Trashing the CO2 hypothesis …

What exactly is a “CO2 hypothesis” ??

March 25, 2013 11:20 am

I’ve said it before and I’ll say it again: the AGW hypothesis is easily and quickly disprovedn by simple observation and even simpler arithmetic.
Observation #1: Temperatures are lower today, 80 years later, than they were in the 1930s despite a 40 percent increase in atmospheric CO2 over that time. Likewise, at least three warm periods, all warmer than today and occurring at times of lolw CO2 concentrations, are well documented in the historical, as well as the physical record: the Hittite-Minoan-Mycenean period, 1800-1400 BC, the Roman Climate Optimum, 100 BC-300AD, and the Medieval Warm Period, 900-1300 AD.
Zero correlation between CO2 and temps for nearly 4,000 years – how much longer a record do we need? (And of course this doesn’t include the end of the last ice age 12,000 years ago, a quite observable length of time before the Industrial Revolution.)
Observation #2: One other, and not the largest other by far, source of CO2 emissions is animal respiration. From respiration rates and volumes and the known percentage of CO2 in exhaled human breath, by simple arithmetic, on average each person on the planet emits about 500 kg of CO2 per year. Extrapolating to the 7 billion of us on Earth, thast’s 3.5 gt per year. As numerous as we are, man is probably less that 1/2 percent of the animal biomass on the planet, and definitely is one of the lower emitters of CO2 on a per-pound of-body-weight basis. But if man is assumed to be an average emitter, this still translates by very simple arithmetic into 700 gt of CO2 from animal respiration. And in all probability that is an understatement, because at least half of the animal biomass is insects, with emission rates per pound of body weight up to 20 times that of humans. This versus 38 gt/yr of gross CO2 from fossil fuels and 10 gt residual, 48 hours after emission.
Observation #3: Water vapor (except in the relatively small areas of super-dry desert on Earth) constitutes anywhere from 30 to 140 times as much of the atmosphere as CO2, figured by simple arithmetic from the vapor pressure of water and the relative humidity. It has a specific heat about 4.5 times that of CO2 – which probably understates the actual coimparative effect of H2o vs. CO2 as a greenhouse gas. But going solely on specific heat, water vapor, by simple arithmetic, has between 135 and 630 times the effect on climate as CO2.nd again this may be a huge understatement.
It doesn’t take a climate scientist to figure these things out, only a reasonably knowledgeable layman who can read and do simple arithmetic. Put simply, man’s activities are an infinitesimal fraction of an infinitesimal factor in climate change, at least four sigmas below the realm of statistical relevance. – mathematically, one over infinity squared.
The alarmists could only get around these facts by destroying every physics and chemistry textbook, monograph and journal article ever published plus all the libraries containing the source documents for the historial record. I doubt that they could do that.

March 25, 2013 11:26 am

@nc – Hope my last post answered some questions for you.

KR
March 25, 2013 11:32 am

Regarding exponential growth in CO2: Obtain the CO2 yearly numbers (http://woodfortrees.org/data/esrl-co2/compress:12 for yearly averages), and drop them into your favorite spreadsheet software. Add a third column which is the log (Ln) of the concentrations, plot it adding a linear trendline.
The log of an exponential is a straight line – the log of the CO2 data curves up faster than a straight line, the concentration increasing faster than exponentially. While the exponent is quite small, it is present, and from the log trend is increasing over time. Hence the forcing of CO2 is increasing just a tiny bit faster than linearly over time.

bones
March 25, 2013 11:40 am

Let me begin by saying that I have been commenting here under the name “bones”, however, my real name is Stan Robertson. I am a retired physics prof, (Ph.D 1969, Oklahoma) having last worked at Southwestern Oklahoma State University. That should take care of the comment of grumpyoldmanatUK.
Before replying to others, I need to correct two minor errors and add a comment. First, the CO2 forcing effects that I calculated for both warming periods should have used the actual number of decades over which the CO2 changes took place, rather than the number of decades required for doubling at the time.
Second, when corrected, the rate of heating at the top of the atmosphere that would have been caused by CO2 would have been only 0.14 watt/m^2 per decade in the 1976-2000 period. This is a good deal less than the 0.25 watt/m^2 per 1.1 decade of the solar cycle at the top of the atmosphere. So dismissing solar effects is completely unwarranted.
Now for some others comments:
@Ed_B: I was only playing the “radiation is everything” game of the warmistas.
: Ethanol was originally used as an ozone reducing oxygenate in gasoline to replace MTBE, which was shown to be both carcinogenic and environmentally persistent.
D. McIntire: The 390/240 ratio is only to roughly account for the effect of the atmosphere lying between the surface and the top of the atmosphere.
Holmes: Amen! and thank you.
@Solomon Green: Beddington is wrong. Surface temperatures respond on a seasonal time scale. The thermal lag is no more than a decade. Thermal lag is the last refuge of warmists who don’t know physics and don’t understand why there has been no warming for the last 16 years.
, Robin Edwards and bobl: Check out this graph of Mauna Loa CO2 data and tell me again that 5% per decade isn’t exponential: http://i1244.photobucket.com/albums/gg580/stanrobertson/TW-co2-t.jpg
@Robin Potter: See my (above) comparison of solar cycle heating with CO2 at the top of
atmosphere.
@rgbatduke: Thank you for your summary! I have enjoyed the things that you post at WUWT.
@Gail Combs and Joe: Thanks for explaining what I was doing!
Tisdale: I sincerely apologize for using your graph without attribution. I have pilfered hundreds of graphics for my own use over the years and I honestly could not remember where I had gotten it. But that was a glaring lapse on my part not to have taken the time to find it. I have enjoyed reading your articles here on WUWT. They have been very informative!
@AlecM: What would be the average (over a year or two) earth temperature as viewed from space in cloudless regions and in the bands of the Atmospheric Window? And what would be the radiation rate in watts/m^2? Would not the average outbound radiation over the entire surface as measured at the surface be about the same?
Lastly, Thanks to the others who took the time to read this.

March 25, 2013 11:44 am

@NC, I agree with you. But it’s even worse because extra CO2 will cause a rise in temperature of exactly 0C! .. nothing, nodda, zilch. All of this conversation is simply mental masturbation, and getting very old.
BTW, belated congrats to Anthony and crew on a marvelous achievement by winning a smashing 3rd Blog award! … that is awesome!

March 25, 2013 11:49 am

Some people think that it has been a good idea to use corn to produce ethanol for a fuel, however, I am not one of them because the energy return on investment is either negative, or minuscule at best. From the standpoint of greenhouse gas emissions, it is a horrendous loser. It may be a biofuel and cleaner burning, might help ameliorate ozone problems and etc, but considering that nearly a gallon of oil is consumed in addition to the gallon of ethanol produced and burned, it is a quantitative loser. (Not that I care at all about the CO2.)

Wildly and completely incorrect. These are not even remotely factual statements. As has been proven and re-proven here many times over.
It currently takes 1 BTU of energy in (be it gasoline, oil, ethanol or other), plus the stored solar value in the feedstock, to create a current average of between 1.6 and 2 units of energy out in the form of ethanol. This is for standard corn ethanol processes. Newer processes including cellulosic biomass are much higher yet.
On being a greenhouse gas emissions loser, another outright false statement. Ethanol use directly reduces greenhouse gas emissions significantly over the use of fossil fuels. The author ADMITS ethanol is cleaner burning and has other environmental benefits. It is a low carbon fuel source and reduces use of fossil fuels – a double benefit to the environment. Whether you CARE about CO2 or not, there is no valid reason not to minimize its production where reasonably possible. AND most importantly ethanol is RENEWABLE,
The ONLY way to make this ridiculous and wholly unsupported claim – that ethanol increases greenhouse gas emissions, is to make the unproven and unsupported claim that growing corn in the Midwest increases corn prices and causes massive reduction in rain forests as they cut them down to plant corn.
Despite a large increase in corn production, and rising prices for many years now – both beginning well before wide use of ethanol – there has been no supportable evidence of this claim. And scientists agree. More than 100 scientists researching biofuel production agree, calling theses claims misguided and based on limited scientific models. “Results from the model have not been verified enough to be useful,” said Harvey Blanch, a professor of biochemical engineering at University California, Berkeley.
http://www.scientificamerican.com/article.cfm?id=biofuel-crops-could-also-cleanse-soil
The facts are that US corn is in most cases far cheaper than locally grown corn. This is exactly the problem in Guatemala – which was held up as one of the poster children for the big, bad ‘ol US corn ethanol industry. Unfortunately for those claims, when one actually bothers to research, rather than blindly repeat partisan talking points, you find Guatemalans could grow plenty of corn. Which they used to do, until imported US corn became so cheap it made more sense to import our cheap corn and grow more profitable crops on their land. Today, Guatemala continues to buy US corn, increasing the amount the ALLOW to be imported, because it is far cheaper than their own, in order to hold DOWN local prices. Oh, and the US has provided ALL the corn Guatemala has asked for.
These oft repeated claims are ignorant in yet another way as well. People in places like Mexico, Guatemala, and other corn consuming regions do not generally eat yellow corn. They eat White Corn – which the US is also a large exporter of. We grow enough white corn to meet our domestic needs and all export demand. It should be noted as well white corn production in the US is a tiny fraction of overall corn output.
When someone makes such easily refuted, incorrect, unsupported and outright specious claims in the opening comments, its hard to take the rest of the article seriously.

Theo Goodwin
March 25, 2013 11:54 am

Stan Robertson, the author, writes:
“Until there is some understanding of the natural causes of these rapid warming periods and their inclusion in the climate models, there is no reason to believe the models. This is simple first year physics.”
There seems to be a tidal movement to embrace “natural variability” and “natural regularities” among once faithful Alarmists. I believe that Beddington appealed to natural variability as an explanation of the last fifteen years of flat temperatures. Recognition of natural variability is a huge step for Alarmists. It was less than two years ago that Trenberth called for a reversal of the null hypothesis – the hypothesis that natural variability explains temperature variation since the Little Ice Age.
Of course Beddington appeals to natural variability in a self-serving way, as an excuse for the modelers, but genuine scientists are publishing articles that attribute most of the twentieth century rise in temperature to natural regularities that are not yet well understood. The future might hold less stonewalling among Alarmists and a greater openness to empirical science.

March 25, 2013 11:59 am

Let’s just cut to the chase:
1-Science gave us pesticides and while science does agree
it’s real “and happening” they don’t agree climate change will be a
real crisis, only that it’s real. I know, strange but true look it up.
2-Science has only said climate change “COULD” be
a crisis and never said it will be. Look it up.
3-Science says comet hits are a real eventual crisis but not
climate change crisis. Look it up.
4-Not one IPCC warning is without “maybe” and
“could be” and….Look it up.
5-27 years of a maybe crisis means it won’t be a crisis.
REAL planet lovers and REAL ā€œprogressivesā€ donā€™t childishly
enjoy loving a planet with fear.

jorgekafkazar
March 25, 2013 12:00 pm

“Thus while CO2 concentration is increasing exponentially with time, the temperature only increases linearly.”
Over the past 20 years, CO2 concentration is increasing linearly, or close enough to it to make little difference:
http://woodfortrees.org/plot/esrl-co2/from:1993/to:2013/plot/esrl-co2/from:1993/to:2013/trend
You can see that any deviation from a constant linear trend is minuscule. There’s a very slight bend in the curve just post Pinatubo (3 or 4 years), otherwise it’s flat as a die at roughly 1.9 ppm/year.

March 25, 2013 12:32 pm

nc says:
March 25, 2013 at 10:36 am
Please someone tell me what I am missing here?
What you are missing is that the 3% human is additional, the 97% natural is mostly back and forth cycling.
Humans add about 3% of the total CO2 input, but don’t contribute to the CO2 output over a year. Thus while 97% of the input is natural, 98.5% of the total input is absorbed by natural sinks. Slightly more sink than source for all natural processes together. The residual 1.5% is what you see as increase in the atmosphere.
It doesn’t make any difference if the natural cycle is 10 times or 100 or 1000 times the human input. As long as the natural inputs are less than the natural outputs, the contribution of nature to the increase is zero and all increase (in total quantity, not in original molecules) is human made.
Think of a factory where you invest each year some money. The turnover of the factory may be huge, 100 times what you invested within a year, but if you see that the factory at the end of the year, every year for the past 50 years or so, shows less profit than what you have invested, it may be wise to look for a better investment…

March 25, 2013 12:36 pm

jorgekafkazar says:
March 25, 2013 at 12:00 pm
See the difference if you plot the CO2 levels from 1960 on:
http://woodfortrees.org/plot/esrl-co2/from:1960/to:2013/plot/esrl-co2/from:1960/to:2013/trend

Jimbo
March 25, 2013 12:46 pm

Am I correct in noting the following?
* The IPCC was founded in 1988.
* The IPCC published its first assessment report in 1990.
* The Atlantic Multidecadal Oscillation (AMO) was discovered in 1994.
* The Pacific (inter-) Decadal Oscillation was noticed in 1997.

Joe
March 25, 2013 12:47 pm

A. Scott says:
March 25, 2013 at 11:49 am
ā€œResults from the model have not been verified enough to be useful,ā€ said Harvey Blanch, a professor of biochemical engineering at University California, Berkeley.
—————————————————————————————————————-
Funnily enough, exatly the same situation we have with all those climate models which that pesky Mother Nature keeps declining to verify.
One model verifying another doesn’t count if they both miss reality btw šŸ˜‰

E.M.Smith
Editor
March 25, 2013 12:48 pm

I understand the desire to “use their rules” to show their claim is not supported by “their facts”; but at least a tiny mention of convection and enthalpy (i.e. evaporation and precipitation) acting as a heat pipe taking heat from the surface to the top of the troposphere (kind of the definition of it, not radiative, convective) would have been nice.
The whole void at the center of the AGW thesis is the ignoring of non-radiative physics.
http://chiefio.wordpress.com/2012/12/12/tropopause-rules/
BTW, folks tend to think of the tropopause as a static lid on the troposphere were radiation has to carry the heat across the barrier. That is “exactly wrong”. There is significant mixing and air flow across it. In reality, it is a Cat 2 hurricane force wind zone where the vertical air flow turns sideways and heads toward the cold pole of the planet (where it makes “Night Jets” and the polar vortex and descends, mixing with stratospheric air on the way). Hardly static.
See this graph of wind speed vs altitude and ask yourself if those wind speeds are static:
http://chiefio.wordpress.com/2012/12/12/tropopause-rules/wind-speed-alt-1090/
That spike in speed is the tropo“pause”
But yes, showing them bogus with their own (broken) world view is a nice touch. Just not the real world…

Jan R. Hansen
March 25, 2013 12:50 pm

Am I missing something here?
What has happened to the established forcing of 3.7 watts/m^2 per CO2 doubling?

March 25, 2013 12:52 pm

Chad Wozniak says:
March 25, 2013 at 11:20 am
Observation #2: One other, and not the largest other by far, source of CO2 emissions is animal respiration.
Right, but irrelevant: alle microbiological, insect and animal breathing simply recycles CO2 into the atmosphere which was captured a few months to a few years before by plants from the same atmosphere. The net contribution of this huge cycle is near zero, but slightly more sink than source.
See: http://www.bowdoin.edu/~mbattle/papers_posters_and_talks/BenderGBC2005.pdf
Fossil fuels were captured millions of years ago in an atmosphere that contained many times the current levels, thus that really contributes to the current atmosphere…

March 25, 2013 12:52 pm

This is an excellent read and I am sure many of you here may have read it before. The section on “Aspergers” and the money available for assistance makes a good parallel to AGW scientists (with apologies to anyone suffering from any sort of similar disease, it is simply meant as an example).
This article discussed when is “Science” science, and when it is NOT science.
AGW —> As some have said here, maybe, just maybe, we don’t know yet.
“Why Science Need Theories”
http://arachnoid.com/theory/index.html

March 25, 2013 1:04 pm

bones says:
March 25, 2013 at 11:40 am
Let me begin by saying that I have been commenting here under the name ā€œbonesā€, however, my real name is Stan Robertson. I am a retired physics prof,…”
Darn it. I had a twenty on the Scottish folk singer.

AndyG55
March 25, 2013 1:16 pm

I do wish people would stop using HadCrud as any sort of reliable temperature record.
We know it has been massively adjusted downwards pre 1980ish. It is only since the satellite era that it holds any semblence to reality.
Someone needs to remove all the adjustments and get back to the real temperature record. That is what should be being used.
I suspect that it is one of the main reason the climate models are so consistently wrong.
They have all been calibrated to this massively adjusted record (and Giss), and therefore have no chance at all of ever being correct even IF the actually did do the right sort of calculations.

March 25, 2013 1:23 pm

Engelbeen –
Actually, you underscore my point, which is that all CO2 activity has no discernible effect of climate, and, as you say, it is therefore irrelevant whether it’s 3.5 gt or 700 gt or whatever. But that’s the lesson the alarmies can’t understand.

Mike M
March 25, 2013 1:25 pm

Philip Mulholland says: “Youā€™ve just generated the perfect mechanism to explain the sudden onset of the next glaciation. /Sarc”
When Al Gore finds out we’ll see photos of polar bears hunting tree frogs in Ecuador.

Jim D
March 25, 2013 1:32 pm

Some mathematical errors there: simply put, a 1.6 W/m2 unbalanced input can change 700 m of water by 0.17 degrees C per decade. CO2 is already well capable of providing that level of forcing, and it is continuous. Doubling CO2 gives more like 3.7 W/m2 (not 2.7).

Chuck Nolan
March 25, 2013 1:35 pm

Stephen Wilde says:
March 25, 2013 at 8:26 am
Ed_B said:
ā€œAfter 5 years of reading here I guess I still donā€™t get it. All that I see is that the earth needs to change its vertical and horizontal heat distribution system a bit. That would mean a tiny bit wider belt of thunderhead clouds, with a slight increase in frequency, a slight increase in ocean surface heat towards the poles, with a tiny increase in temperature at the polesā€
Agreed. Have been saying as much for years.
—————
That’s what I thought, too!

thingodonta
March 25, 2013 2:50 pm

The official position of the Australian Academy of Science is that they have no idea what caused the warming in the first half of the 20th century. Its in their ‘facts’ about global warming on the website. They say the sun couldn’t have done it, and neither could C02. They then state that if there was some mechanism that amplified the solar effect during this period, then we don’t yet know what it is….which leaves the obvious question, that if there was an unknown amplifying mechanism for solar effects in the first half of the 20th century……what about the second half??? Oh yes, but the second half was all C02….
Completely daft I say.

March 25, 2013 3:00 pm

squid2112 says:
March 25, 2013 at 11:09 am
vukcevic says:
March 25, 2013 at 9:28 am
Trashing the CO2 hypothesis ā€¦
What exactly is a ā€œCO2 hypothesisā€ ??
ā€¦ā€¦ā€¦ā€¦
Not something I have either the time, inclination or desire to find out.
I am interested in the ground level (ground is the operative word here) reality not a fantasy.

bones
March 25, 2013 3:46 pm

Jim D says:
“Some mathematical errors there: simply put, a 1.6 W/m2 unbalanced input can change 700 m of water by 0.17 degrees C per decade. CO2 is already well capable of providing that level of forcing, and it is continuous. Doubling CO2 gives more like 3.7 W/m2 (not 2.7).”
I agree that CO2 doubling gives 3.7 w/m^2, but as I stated, when corrected for the aerosols (plus other GHGs), it yields the net 2.7 w/m^2 of the IPCC.
It is also true that a NET 1.66 W/m^2 could warm sea water by 0.174 C/decade, HOWEVER, that would not describe the situation of either rapid warming period, both of which started from conditions of stable temperature with a ZERO net heating rate. As heating begins, the mixing zones of the upper 50 meters or so of oceans begin to respond in only a few months. Thus the temperature rise starts from a zero heating net rate and ends at the rate at the end of the decade. In order to average the 1.66 W/m^2 plus the surface radiation losses, the heating rate must reach the 4.3 W/m^2 that I calculated.

alex
March 25, 2013 4:00 pm

Citation
——————-
According to the calculations of the UN IPCC, a doubling of the atmospheric concentration of CO2 (with an accompanying rise of other greenhouse gases) would reduce the outgoing infrared radiation from the earth by a net 2.7 watt/m^2 at the top of the atmosphere. This is known as the ā€œclimate forcingā€ that will occur along with a doubling of the CO2. This is a relatively straightforward, but messy calculation. I have repeated the IPCC calculation for CO2 and … we agree. It is important to note that the surface temperature increase that will accompany the CO2 is proportional to the logarithm of the CO2 concentration.
——————
Frankly speaking, I do not quite understand these “messy calculations”.
Because apparently you repeated them, I’d like to ask a few questions.
1. Which spectral properties of CO2 did you assume? The fact is, at the 1 Bar pressure, CO2 has an absorption band in the infrared, Thus, CO2 is a greenhouse gas.
2. When the pressure drops, the CO2 absorption band dissolves into a set of individual absorption lines. CO2 is not a greenhouse gas anymore at low pressures. Did you include this effect into your calculations?
3. Because of (2), the hight where the IR freely leaves the Earth should not depend on the CO2 concentration, but is fixed. It is the pressure level, where the CO2 IR absorption band dissolves.
4. Because of (3), there should be essentially no temperature dependence on the CO2 concentration in the atmosphere at all (at the present level). Not even logarithmic.
Am I wrong?

Lars P.
March 25, 2013 4:08 pm

Thank you Stan!
Have not found yet such a concise and clear description of the CO2 effect and calculation.
Ah, always a pleasure to meet good, solid physics calculations!

Alvin
March 25, 2013 4:30 pm

“This is simple first year physics.”
This is the point. What were many/most of the climate alarmists doing that year of organized instruction? Marketing, creative writing, or global studies most likely.

george e. smith
March 25, 2013 4:52 pm

Well, I think Professor Robertson’s essay has its own myths that should be dispelled.
I first balked at the internal combustion / electric motor comparison. Depending on application, that swap may be a very good idea. For example, if you are pumping water to irrigate, using a diesel engined pump, a swtch to electric would be a very good idea. Some CA central valley ag farms, do use diesel pumps. And keeping a supply of diesel around is a pain. Electric motors are very efficient, compared to diesels, and the instant on/off feature, makes them a lot more attractive, when you have convenient electricity. Yes the electricity may have been expensive to get from other energy sources, but once you have it, it can be used very efficiently. But for driving your car a nyet on that.
As for myths, there’s that one about an exponential growth of CO2. Mauna Loa data certainly doesn’t show that.
And the myth about earth surface temperature going as the log of CO2. Not a shred of data establishing that myth. Right now, CO2, continues to grow, and Temperature does nothing.
No data shows a log relation as any more likely than linear, or exponential; I like fitting the data to the function:- y = exp (-1/x^2) as my personal best fit function.
Also, the primary source of radiant cooling of the earth, is the planet’s hottest deserts, in daytime. They radiate at about double the rate of the IPCC’s silly average 390 W/m^2. Watts is a unit of power, and Watts per m^2, a unit of power density.
Power is an instantaneous quantity. There is no such thing as average power, or RMS power either.
The “average” power level of Hurricane Sandy, was barely noticeable given the frequency of occurrence. So why are all those East coasters complaining about damage. On average, the damage will be barely noticeable compared to the ravages of time.
You DO NOT average power; on average, nothing happens.

Niff
March 25, 2013 5:04 pm

People, Stan has rattled several cages. Cool. But he DOES know numbers….http://journalofcosmology.com/RobetsonLeiter.pdf
I puzzled for a while over exponential Co2….note to self…pay attention.

Niff
March 25, 2013 5:05 pm

Thanks for joining us Stan….looking forward to your next comment!

george e. smith
March 25, 2013 5:18 pm

I was interested in the “first year physics” mention; although surprised at the lack of any mention just what “physics” that might be.
I actually taught “first year physics” a half century ago, so maybe the physics was different then. Well more accurately that would be first year university physics. Now when I myself got to “first year physics (university), I had already had five full years of physics. I just went to a different high school than most people.
And that “first year physics” that I taught was actually first year pre-med physics, for would be doctors and veterinarians (200 of them that year). And I only taught part of the first year pre-med physics; just the geometrical optics, and atomic physics portion.
So I don’t know if any of that would invalidate IPCCmodels of global warming.

Mario Lento
March 25, 2013 5:47 pm

Uhm… CO2 is NOT increasing exponentially. The correct phrase should be CO2 is increasing linearly (mostly naturally but some probably anthropogenically) and the effect of the linear increase is a logarithmic increase in the effect of CO2 as a greenhouse gas. That is of course, if we assume that CO2 is in fact a greenhouse gas. Certainly it has an effect on the lapse rate of heating and cooling!!!

ferd berple
March 25, 2013 5:48 pm

Ed_B says:
March 25, 2013 at 4:46 am
I stopped reading right here. WTF?
========
Too bad, you missed the best part:
“These straightforward calculations make it painfully obvious that CO2 forcing is not what drove the two periods of rapid heating during the last century. Until there is some understanding of the natural causes of these rapid warming periods and their inclusion in the climate models, there is no reason to believe the models. This is simple first year physics.”
Unfortunately first year physics is not a prerequisite for Climate Science.

ferd berple
March 25, 2013 6:05 pm

Robin Edwards says:
March 25, 2013 at 8:30 am
Can someone please sort me out?
Robin
==============
CO2 is increasing exponentially. A linear relationship is also increasing exponentially, so long as you use 1 as the exponent. The question is, how much greater than 1 is the exponent? If the answer is “not much” then a linear relationships will be pretty accurate unless one is making voodoo predictions 100 years in the future.

ferd berple
March 25, 2013 6:07 pm

george e. smith says:
March 25, 2013 at 5:18 pm
So I donā€™t know if any of that would invalidate IPCCmodels of global warming.
==========
the part that happens when you apply it to the data.

Chris Edwards
March 25, 2013 6:25 pm

Re A Scott, ethanol is a poor substitute for fossil fuels, it increases fuel consumption and degrades many components in the fuel system. Stihl will not cover ethanol damage to their machines in the warranty and IMHO quite right too! I produces less horsepower so degrades the driving experience of the vehicle, this is leaving aside the morality of starving the real poor to produce a poor quality fuel that few would buy if the did not have it forced on them.

Mario Lento
March 25, 2013 8:39 pm

Chris Edwards says: “I produces less horsepower so degrades the driving experience of the vehicle” I assume you mean ethanol produces less horsepower.
What I think you mean is that it is less dense of a fuel. That is, a gallon of ethanol will drive a car about (I’m guessing here) about 70% of the distance of a gallon of gasoline. However, converting a car’s engine to ethanol will enable the engine to make significantly more power (you need bigger injectors). What happens is that the extra fuel (and it’s higher latent heat of vaporization) cool the combustion chamber such that much more aggressive tuning can be had. I’m pretty sure converting an engine to ethanol can get about 30 to 40 % more power than running it on gasoline.
Think – 800hp 2.4l turbo engines on track day cars…
Still: It’s a bad idea to take perfectly good drinking alcohol and use it for transportation šŸ™‚

AndyG55
March 25, 2013 8:47 pm

KR says ” Hence the forcing of CO2 is increasing just a tiny bit faster than linearly over time.”
So still basically zero. šŸ™‚

Jim D
March 25, 2013 9:29 pm

The manmade part of CO2 is increasing exponentially. That is, if you subtract 280, what is left is exponential with a 33-year doubling time to a good approximation through the 20th century.

AlecM
March 26, 2013 12:01 am

bones: ‘@AlecM: What would be the average (over a year or two) earth temperature as viewed from space in cloudless regions and in the bands of the Atmospheric Window? And what would be the radiation rate in watts/m^2? Would not the average outbound radiation over the entire surface as measured at the surface be about the same?’
Look at Hansen et. al. 1981 paper and following Houghton they claim the GHE = 33 K by attributing it all to the effect of water on reducing lapse rate hence raising the tropopause. This is very difficult to disprove because to do so you have to show definitively that increase in [CO2] does not increase [H2O].
I have however done it by using my process engineering training, but my PhD was applied physics. There are 13 mistakes in climate alchemy. the heat transfer is the worst. The main GHGs switch off IR emission in their bands. If they did not there would no radiative thermal equilibrium and we’d be a thin wisp of gas. The reality is that the GHGs warm the surface. There is no GHG blanket. The forcing argument is stupid. There can be no direct thermalisation at LTE.Direct thermalisation at clouds is a cooling mechanism.
So in answer to your question, there are a large number of control systems for the planet. There is no average temperature unless one uses a GHC with the correct physics to establish it. The main parameters are AW radiation and evapo-transpiration limiting upper temperature in cloudless regions during the day. At night condensation controls temperature. So the AW and the water cycle are the most important part of the planetary temperature control. Then we have clouds which restrict the AW heat loss and further reduce diurnal temperature range.
But as we are seeing with the present cold weather, the jet streams are moving nearer the equator because solar EUV is low and the low magnetic field of the sun has increased cloud cover. This means the Hadley cells are shorter and the planet is cooling in the Northern land mass. Very soon the seas will cool in the cooling ENSO; we are heading to 17th Century weather. There will be mass famine in some years.
CO2 will equilibrate at ~450 ppmV as the seas cool and the vegetative growth rate rises sufficiently.

AlecM
March 26, 2013 12:03 am

Indirect thermalisation at clouds

AlecM
March 26, 2013 12:11 am

Sorry GCM not GHC.

March 26, 2013 2:28 am

Chris Edwards says: March 25, 2013 at 6:25 pm
Re A Scott, ethanol is a poor substitute for fossil fuels, it increases fuel consumption and degrades many components in the fuel system. Stihl will not cover ethanol damage to their machines in the warranty and IMHO quite right too! I produces less horsepower so degrades the driving experience of the vehicle, this is leaving aside the morality of starving the real poor to produce a poor quality fuel that few would buy if the did not have it forced on them.

Like the original author your claims are also false. You recycle years old claims that are not accurate or correct.
Ethanol does have fewer BTU and thus gets slightly reduced mileage. Using the current E85 blend there is roughly 20% lower MPG. E85 also costs less, meaning the cost per mile is almost identical. It is an excellent replacement for fossil fuels, burning cleaner and reducing emissions, plus it is a RENEWABLE energy source.
Every manufacturer supports E10 blends, and they will not harm any reasonably newer vehicle. If you have a 70’s muscle car or an old boat with a fiberglas tank you probably shouldn’t use ethanol. You should not use ethanol if you have a 20 year old chain saw, mower or other ancient gas powered equipment.
Stihl is perfectly happy to have you use ethanol in their products. So does Husqvarna, among others.
Ethanol and ethanol blends have HIGHER octane, and thus run better, not worse than gasoline. I have a 2003 bi-fuel Tahoe. I run E85 appx. 50% of the time. My cost per mile was generally lower using E85 before the ethanol tax credits expired. Now it runs a few percent higher or lower depending on the market and gas prices. It does absolutely nothing to “degrade” the driving experience whatsoever – and that is based on 100,000 miles of E85 experience over a number of years.
And your “starving the poor” claim has repeatedly been proven false. As I noted above, US corn exported to Guatemala is not starving the Guatemalan’s because if its high price – to the direct contrary, Guatemala’s government purchases US corn becasue it is significantly CHEAPER than their own – they buy US corn (and we provide everything they have asked for) to LOWER the cost of corn for their citizens.
There is nothing “poor quality” about ethanol. People need to educate themselves and stop posting these false and refuted old claims.
Ethanol is not, and never will be, a full replacement for fossil fuel – at least not using corn. It IS an excellent and necessary “bridge” – reducing fossil fuel use now, which extends our fossil fuel reserves and reduces foreign energy dependance – while helping develop the vehicle and distribution base necessary to support future, much more efficient, renewable ethanol processes and sources.

trevorH
March 26, 2013 2:31 am

its been the coldest March in the UK for 50 years more snow than 30 years. the govt chief scientist witters on that its extreme weather caused by climate change — !!!!
The BBCs own forecasters have to admit its because of winds fro the east.
‘Scientists!!’ do they exist any more?

richard verney
March 26, 2013 3:12 am

My understanding is that racing cars of the past used to run on an ethanol mix because more bhp could be extracted. ie., they went faster. No doubt, they were not worried about economy. Nor were they worried about CO2 emissions.
Whether there is a net reduction in CO2 emissions by using some form of ethanol mix is a moot question. One has to take into account the CO2 produced in producing the ethanol, and the fact that the fuel economy of an ethanol mix is lower so that overall more fuel is burnt per mile.
Overall, it is doubtful that there is any net reduction of CO2 in using biofuels/ethanol mixes.

rgbatduke
March 26, 2013 3:45 am

Anyone who accepts that the Earth emits real radiative energy at the black body flux which assumes a sink at absolute zero, is a lunatic. This makes all climate alchemists lunatics. They believe that theyā€™re right because pyrgeometers measure temperature then convert it by the S-B equation to Power, also the potential flux to a sink at absolute zero. This is also lunacy but includes the meteorologists as well!
This is a straw man argument, as what is accepted is an entire measured infrared spectrum of outgoing radiation incident on a night sky that, except for two tiny patches covered by the Sun and moon, is at an almost uniform temperature of 3K, close enough to absolute zero (compared to the temperature of the Earth) as to make no difference. The Earth, without any question whatsoever, emits real radiative energy into a sink at (almost) absolute zero. In fact, every day, it emits almost exactly as much real radiative energy as it absorbs from the Sun (plus a teensy bit from other sources) and hence remains in a very approximate dynamical equilibrium. The evidence is a mix of nearly complete understanding of the physics involved in this process plus direct photographs of the spectrally decomposed outgoing radiation. It is not the classic blackbody radiation curve because the emission doesn’t come from a homogeneous black body, but one can discern consistent in-band blackbody radiation from the surface at its approximate temperature within that curve from unblocked parts of the spectrum.
There are several places you can look at those IR spectrographs — they were republished on WUWT in at least one article on the greenhouse effect, they are in Grant Petty’s A First Course in Atmospheric Physics, selected ones are available on the internet in a variety of places. In the meantime, asserting that the Earth — a continuously warmed object floating in a vacuum — doesn’t cool radiatively — that really is lunacy.
On a separate note: Watts is a unit of power, and Watts per m^2, a unit of power density. Power is an instantaneous quantity. There is no such thing as average power, or RMS power either.
I sometimes wonder, are the people who post this sort of thing really as ignorant as they appear to be? Watts are indeed Joules per Second in SI units. Watts per meter squared are units of intensity (not usually expressed as “power density”) and this is a quantity that is most meaningful (in the case of electromagnetic radiation, which is directional) when expressed as the Poynting vector \vec{S} = \frac{1}{\mu} (\vec{E} \times \vec{B}). Emitted or absorbed radiative power (through/by a surface) is technically the flux of the Poynting vector. Finally, there is, in fact, such a things as average power: P_{av} = \frac{1}{T} \int P(t) dt and in the case of electromagnetic power associated with a harmonic wave the average power is 1/2 of the peak power (from averaging e.g. \sin^2(\omega t)). Technically, this means that there is a RMS amplitude to the electric or magnetic field that leads to the average power, but if one referred to RMS power I would certainly understand that one meant average power as opposed to instantaneous power (where there is no “root mean” derived from the square).
rgb

richard verney
March 26, 2013 4:05 am

A. Scott says:
March 25, 2013 at 11:49 am
////////////////////////////////////////////////////////
The price of US corn/grains is only one factor and the issue is not whether (due to efficiencies) the US can produce grain at a cheaper price than some foreign country can do so through its own domestic production.
Some suggest that the increase in grain price was one factor behind the Arab Spring which, in itself, has led to thousands of deaths. So political commentators consider that there has been an increase in the price of export grains
One important factor not addressed by you, is surplus quantity. How much grain does the US export (say over the past 30 years), and much grain is exported in it food aid programme (say over the past 30 years)? A sub issue is the use of GM crops to try and make good the shortage of conventional crops which shortage has been caused by the switch in land use to biofuels.

richard verney
March 26, 2013 4:17 am

AndyG55 says:
March 25, 2013 at 1:16 pm
//////////////////////////////////////////////////////
But your observation applies to all the data sets (with the exception of the satellite data set which data set has its own problems and is of a rather short duration).
Where can one find a data set comprising of raw data produced only from good sited stations with no station drop outs or moves?
People have no choice but to use the basterdised data sets since this is all that is readily available.

March 26, 2013 4:35 am

Ferdinand Engelbeen:
Concerning observed atmospheric CO2 increase, at March 25, 2013 at 12:32 pm, you yet again make the unfounded and implausible assertion

the 3% human is additional, the 97% natural is mostly back and forth cycling.

Please desist from asserting your improbable assumption as fact.
It is extremely unlikely that ā€œthe 97% natural is mostly back and forth cyclingā€ which can be assumed to be constantly in balance. This improbable balance may exist, but nothing else in nature is observed to be so in balance and constant.
An imbalance of less than 2% p.a. between the natural emission and sequestration would account for all of the observed rise in atmospheric CO2. And there are several possible reasons why such an imbalance may have occurred.
One example of such a possible cause of natural altered imbalance is variation to undersea volcanism centuries in the past. Increased undersea volcanism would release additional sulphur ions which travel with the thermohaline circulation until they reach ocean surface layer centuries later. The increased sulphur in the ocean surface layer would reduce the pH of the layer with resulting alteration to the equilibrium concentrations of CO2 in the air and ocean surface layer.
A change of only 0.1 in ocean average pH (which is much, much too small for it to measurable) would induce a rise in atmospheric CO2 concentration which is larger than has been observed in the past century. Hence, undersea volcanism could be the cause of ALL the observed rise to atmospheric CO2 concentration. And there are other possible causes, too.
Please note that the human emission of CO2 would not affect this in any way: the changed equilibrium would be the same whether or not the human emission existed.
And you are plain wrong when you repeatedly assert

It doesnā€™t make any difference if the natural cycle is 10 times or 100 or 1000 times the human input. As long as the natural inputs are less than the natural outputs, the contribution of nature to the increase is zero and all increase (in total quantity, not in original molecules) is human made.

It matters a very, very great deal because the improbable assumption of balance between the natural inputs and outputs provides the conclusion that emissions from humans are causing the rise in atmospheric CO2 concentration. And that unfounded conclusion is distorting energy and economic policies around the world.
I donā€™t know if the rise in atmospheric CO2 concentration is entirely natural, or entirely anthropogenic, or partly natural and partly anthropogenic. But I want to know.
The myth that natural emissions and sequestrations of CO2 are known to be in a constant balance needs to be dispelled if we are to determine the true causes of the recent rise in atmospheric CO2 concentration and, thus, to avoid distortion of energy and economic policies.
Richard

Chris Wright
March 26, 2013 5:06 am

StephenP says:
March 25, 2013 at 5:13 am
“I see that we are all doomed, according to the outgoing chief government scientific advisor, Sir John Beddington:….”
I heard him on the Today program yesterday. Once again the BBC is listening to the doom mongers while shutting out any sceptical opinions based on actual data. Of course, he didn’t mention any inconvenient facts e.g. the lack of global warming for getting on for two decades and that the English climate has been rapidly and consistently getting colder since 2000, as shown by the CET.
Before rapidly switching channels, I glimpsed him again on the BBC news last night. He said something like: “We know the weather will be more extreme….”
We know? Does he have a crystal ball? What about all the uncertainty in climate science? What about all the forecasts by the climate models that have turned out completely wrong? What about the missing tropical hot spot, and the fact that the world’s IR emissions are actually increasing as CO2 increases, in direct contradiction to the dogma (Lindzen)? Are these morons completely incapable of learning anything? Or are they simply liars who, deep down, know the game is up but they have their careers and pensions to think of?
And yet he has the gall to say “We know…”
I find it completely sickening.
Chris

Richard Poor
March 26, 2013 5:11 am

Endless argument and politics about CO2 in most discussions ignore the amount of money involved or the toxins evolved. The production and combustion of ethanol adds aldehydes and ketones to the sidewalk environment. The extraction and combustion of coal adds <2.5 micron particulates proven to be harmful. Then there is Hg and other metals and acid mine waste water. Disposal of nuclear waste is and has been a problem. Nuclear weapons proliferation is a problem. Nuclear accidents that are supposed to be so rare as to not amount to concern gave rise to Cherynoble and Fukushima.
Charging plug in hybrid vehicles with solar electricity eliminates range anxiety, bridges the fossil fuel aftermarkets and could eliminate 2/3 of the need for global oil and oil related pollution. But oil profit would diminish as well. Efficiency, appropriate energy conserving and solar utilizing architecture, improved insulation and fenestration all contribute to more comfort and less pollution but diminish fuel vendors profits.
The issue as I see it is not CO2 but what price for pollution and nuclear risk? Significant global cooling would exacerbate the levels of pollution as would significant global warming. Staying comfortably and artificially warm or cool in homes, work places and vehicles requires energy.

James
March 26, 2013 8:20 am

K.I.S.S. Not the band the principle of Keep It Simple Stupid. The issue is funding. Scientists need it to pursue knowledge but funding is limited. If you don’t publish you get no funding, if you publish something boring you may get some funding, if you publish something that excites people you get lots of funding. What’s more exciting than yelling “we’re all going to die”? Institutions push for exciting research (they need money too), and the Government pushes for answers to the “Important Questions” (cynics can read that as issues I can get elected/re-elected on) without understanding that the aquisition of knowledge often procedess at its own pace.

March 26, 2013 9:19 am

Sounds familiar
http://www.vukcevic.talktalk.net/NP.htm
but it is 1940 news
(Real-science.com & Tallbloke’s Talkshop)

Lars P.
March 26, 2013 10:17 am

AlecM says:
March 26, 2013 at 12:01 am
…But as we are seeing with the present cold weather, the jet streams are moving nearer the equator because solar EUV is low and the low magnetic field of the sun has increased cloud cover. This means the Hadley cells are shorter and the planet is cooling in the Northern land mass. Very soon the seas will cool in the cooling ENSO; we are heading to 17th Century weather. There will be mass famine in some years.
CO2 will equilibrate at ~450 ppmV as the seas cool and the vegetative growth rate rises sufficiently.

Probable scenario, and due to the CAGW-chicken-little&fanatics this is not being accounted for and we go totally unprepared for such scenario if this happens.
richardscourtney says:
March 26, 2013 at 4:35 am
Ferdinand Engelbeen:
……
I donā€™t know if the rise in atmospheric CO2 concentration is entirely natural, or entirely anthropogenic, or partly natural and partly anthropogenic. But I want to know.
The myth that natural emissions and sequestrations of CO2 are known to be in a constant balance needs to be dispelled if we are to determine the true causes of the recent rise in atmospheric CO2 concentration and, thus, to avoid distortion of energy and economic policies.

Thanks for pointing that out. There is still a lot to learn about the carbon cycle and so many uncertainties, as example the undersea vents and volcanoes which cover 70% of the surface of the globe, but due to the much thinner earth-crust should represent a higher percentage then 70% of total emissions:
http://www.newscientist.com/article/dn12218
“The team estimates that in total there could be about 3 million submarine volcanoes, 39,000 of which rise more than 1000 metres over the sea bed.”
Also the quantities that are absorbed by vegetation are being re-estimated:
http://www.sciencedaily.com/releases/2003/05/030509084556.htm
“Why would a forest grow so well on arid land, countering all expectations” – I chuckle when I see such comments.
Phytoplankton too:
http://climaterealists.com/index.php?id=9768
“This wasn’t just any phytoplankton bloom,” Stanford University marine scientist Kevin Arrigo told The Christian Science Monitor. “It was literally the most intense phytoplankton bloom I’ve ever seen in my 25 years of doing this type of research.”

bw
March 26, 2013 12:15 pm

It’s heartening to see bloggers adding sanity to the “AGW” issue.
Chad, Ed_b, EMSmith, AlecM, george e. and others deserve credit for taking the time to respond to Stan/Bones.
I’ll add that the carbon cycle has both biological and geological properties, with enough convolution to make oversimplification a constant danger. Biology/ecosystems do not often lend itself to linear/physical analysis. Thus the danger of “averaging” in an attempt to simplify the issues to tractable understanding.
Is there an “average” boundry layer? “Average” surface albedo? Average desert?
Average global temperature? Average carbon flux? Obviously not.
Before fossil fuels, were the global carbon sources to the atmosphere at 100 percent? The global carbon sinks were 100 percent? Well yes. But both sources and sinks obviously vary.
A billion years of variable biology has caused the atmosphere to have variable carbon sources and variable carbon sinks. Makes sense that the 100 percent carbon sinks will accomodate the added fossil CO2. That is, the gigatonnes of fossil CO2 has increased the total carbon cycle to 103 percent of the pre-industrial levels. The atmosphere is not a stagnant pool, the carbon cycle will adapt to about 103 percent of the pre-industrial fluxes.
C4 photosynthesis evolved as a response to vitally low CO2 levels, rest assured that the ecosystems will adjust as needed. Time is on our side. Best to stop feeding the insane “alarmists” with money.

george e. smith
March 26, 2013 12:18 pm

“””””……rgbatduke says:
March 26, 2013 at 3:45 am
…………………….
On a separate note: Watts is a unit of power, and Watts per m^2, a unit of power density. Power is an instantaneous quantity. There is no such thing as average power, or RMS power either.
I sometimes wonder, are the people who post this sort of thing really as ignorant as they appear to be? Watts are indeed Joules per Second in SI units. Watts per meter squared are units of intensity (not usually expressed as ā€œpower densityā€) …….””””””
Some of them actually may be.
Actually “Intensity” or “radiant intensity” as it applies to electro-magnetic radiation has SI units of Watts per steradian.
Never is it Watts per metre sqared.
I used ” power density” as a shortened form of “areal power density” meaning the rate at which energy passes through a cross sectional area perpendicular to the direction of propagation, antwhere in the EM field; and those SI units are properly “Watts per metre squared” or “Joules per second per metre squared” if one prefers.
And I deliberately used that form to distinguish that case from the alternative cases of “Watts per metre squared” which also are the proper SI units of “radiant emittance” when referring to radiating surfaces, and “irradiance” when referring to radiant energy receiving surfaces.
But in any case, what is measured is the instantaneous value, that depends on the square of of the electric field (at that location).
Since we are taught that all objects above zero Kelvins Temperature must radiate EM energy at a rate (radiant emittance) depending on that Temperature, that object can only rise in Temperature, (due to incident radiant energy), if the (instantaneous) rate of incident energy absorption; (irradiance x absorption coefficient) exceeds the pre existing emission rate.
Whether the object heats or cools (radiatively), depends only on whether it gains or loses energy (time integral of the power). The maximum Temperature it can reach, depends on the instantaneous rate at which absorbed energy exceeds radiated energy (assuming radiation is only operating energy transport process).
But “intensity” or “radiant Intensity” is a quantity that pre-supposes a “point source”, since its SI units are “Watts per steradian” and not Watts per metre squared; points having zero area.
Practical measurements of “Intensity” require that the source-sensor distance exceed ten times the (real non-point) source diameter. At such a distance, the error is about 1/2% due to the source size.

george e. smith
March 26, 2013 1:43 pm

“””””…..rgbatduke says:
March 26, 2013 at 3:45 am
Finally, there is, in fact, such a things as average power: and in the case of electromagnetic power associated with a harmonic wave the average power is 1/2 of the peak power (from averaging e.g. ).
.(……plus stuff that doesn’t cut and paste……. )
And it is every bit as real, as is the average telephone number in the Manhattan telephone directory.
Averaging is a well defined (and fictional) result of performing an algorithm on a set of numbers, which themselves may represent actual things or values, that could have been actually observed, or measured, by someone, in an actual real situation or experiment for example. In that sense, the original numbers can be described as “real” in that they represent something that actually was observable. But the process (averaging) is just as valid, when applied to an entirely ad hoc set of numbers that are unrelated to each other in any way.
ALL of these real observable numbers, are replaced by the algorithmically defined number, to give an entirely fictional number, that was not, and can not, ever be observed by anybody, or any thing.
In that sense, they (averages) do not exist.
Averaging, and averages, are defined elements of the discipline of mathematics; specifically, that branch of mathematics, called statistics. Within that discipline they can be regarded as real, being defined operations valid within that discipline.
In the real world, where observations are made, or experiments conducted, and measurements taken, averages are not observable, nor do they have any influence on the outcome of an experiment. Physical systems only respond to real time actions and phenomena.
One can manipulate the data of any real world experiment, in any manner of fashion one desires, according to any algorithm, one wants to describe, and for any reason. The results of that process are properties of the algorithmic process; they are not properties of the real world system being observed.

March 26, 2013 3:48 pm

richardscourtney says:
March 26, 2013 at 4:35 am
The myth that natural emissions and sequestrations of CO2 are known to be in a constant balance needs to be dispelled if we are to determine the true causes of the recent rise in atmospheric CO2 concentration and, thus, to avoid distortion of energy and economic policies.
Dear Richard,
We haven been there for several years now, but for new readers not familiar with our discussion, I will give my opinion again in short:
While I agree with you that the rise of CO2 shouldn’t be used to distort our energy supply and economics, I strongly disagree with the first halve of what you say, as the natural emissions and sequestrations of CO2 are near in balance over the past 50+ years. With slightly more sequestration than emission. That is simply calculated from the changing mass balance over the years.
Every year the inventory of the human emissions is made from fossil fuel sales (taxes!) and burning efficiency. Maybe a little underestimated, but fairly good. Every year the average increase of CO2 in the atmosphere is measured to a high degree of accuracy. The difference between these two is what nature has done in the past year: more sequestering than emissions or the opposite or no change. It doesn’t matter at all if some volcano exploded that year or a lot of forests were burning down or seawater temperatures increased a lot, the net result of all these individual events together is known for every year in the past 50+ years with a reasonable degree of certainty. And that shows remarkably little variation (+/- 2 GtC or +/- 1 ppmv) over the years. Here the graph:
http://www.ferdinand-engelbeen.be/klimaat/klim_img/dco2_em.jpg
It is quite simple: as long as no CO2 is escaping to space (which is the case for hydrogen…), the mass balance shows that nature as a whole is a net absorber for CO2 over the past 50+ years, not a net contributor. No matter if the natural cycle was 10 or 100 or 1000 GtC/year into and 14 or 104 or 1004 +/- 2 GtC out of the atmosphere.
Further, all available evidence agrees with human use of fossil fuels as the cause of the increase, every alternative I heard of violates one or more observations. That includes undersea volcanoes and ocean acidification…
See further:
http://www.ferdinand-engelbeen.be/klimaat/co2_measurements.html#The_mass_balance

Gail Combs
March 26, 2013 4:42 pm

All of Ferdinand Engelbeen’s information is based on the hypothesis conjecture that CO2 is well mixed in the atmosphere. Data is tossed if it does not met the “well-mixed’ assumption. This is a fundamental underlying assumption to the CAGW scam. With out this conjecture the whole edifice falls apart.
Several people have shot down this conjecture.
Lucy Skywalker gathered together much of that information here.
Jeffrey A. Glassman, PhD wrote two very good in-depth science articles on the subject
THE ACQUITTAL OF CARBON DIOXIDE
ON WHY CO2 IS KNOWN
NOT TO HAVE ACCUMULATED IN THE ATMOSPHERE &
WHAT IS HAPPENING WITH CO2 IN THE MODERN ERA

And finally a newer article by E.M. Smith looking at the actual data in CO2 global maps that anyone can understand here
The oceans cover 70% of the earth’s surface. The ocean temperatures change, the amount of green growing stuff changes, the Jet stream and trade winds change, the amount of CO2 put out by humans and volcanoes varies yet we are to accept the fact that CO2 is well mixed….

March 26, 2013 6:27 pm

richard verney says: March 26, 2013 at 4:05 am

One important factor not addressed by you, is surplus quantity. How much grain does the US export (say over the past 30 years), and much grain is exported in it food aid programme (say over the past 30 years)? A sub issue is the use of GM crops to try and make good the shortage of conventional crops which shortage has been caused by the switch in land use to biofuels.

I have addressed this question regularly. Anyone can go to the US Field Grain Yearbook (or the extensive other data the USDA provides) and find out the true facts instead of repeating partisan rhetoric.
As to corn – contrary to the claims that ethanol production has reduced or negatively affected corn exports:
*Total US Corn exports averaged 47 million metric tons a year fro period 1991-2011. The average US corn exports for the peak ethanol increase years of 2002 – 2007 was 51 million metric tons with a top number of 6o.7 million metric tons in 2007/08.
*During the period since corn use began increasing due to ethanol – 2002 thru 2011 – US corn use for fuel increased from 997 to to 5,018 million bushels. Yet the US exports increased significantly during the height of that period.
*Prices both increased AND decreased significantly from 2002-2011, despite corn used for ethanol growing substantially.
*The US has met ALL domestic use, both food and fuel, met ALL export demand, AND has still maintained an average annual surplus of 1.4 billion bushels during the period 1991-2011

The price of US corn/grains is only one factor and the issue is not whether (due to efficiencies) the US can produce grain at a cheaper price than some foreign country can do so through its own domestic production. Some suggest that the increase in grain price was one factor behind the Arab Spring which, in itself, has led to thousands of deaths. So political commentators consider that there has been an increase in the price of export grains

More unsupported allegations. The US has provided, during the 1991-2011 period, up to 92% of Egypt’s corn imports – on average over 71% each year during this entire period.
The truth is in the data. While US corn exports declined slightly from 2008 – from an almost exactly average year of 47,900 million metric tons to appx 39,000 million metric tons in 2011, its is becasue the other corn exporting country’s – Argentina, Brazil, India and the EU combined, increased their corn exports from 16,900 to 52,200 million metric tons from 2005 to 2011. Their cheaper corn has reduced the overall export demand in the US.
And what about your claims about prices? Again the data tells the truth.
*1971-1991 Corn prices averaged $2.28 a bushel, with prices mostly between $2.50 and $3.20 for that period.
*1991-2011 Corn prices averaged $2.57 a bushel, with prices mostly between $1.85 and $3.04 for that period.
*2001-2011 Corn prices averaged $3.37 a bushel, with prices mostly between $1.97 and $3.55 for that period.
There are short, speculator driven spikes during each of these periods, as with today. But the reality is, absent those short spikes, Corn prices today are very little changed than they were in the 1970’s
Regardless – lets take the 1971-1991 average of $2.28 a bushel and compare to the 2011 price of $5.18. That would be a 127% increase from 1971 to 2011, a 40 year period. That would be an average annual increase of just over 3%.
From 2001 to 2011 the US supplied 0.566 billion metric tons of corn exports comprising 52% of ALL world corn exports.Most of that corn was supplied at the same prices as corn sold for in the 1970’s.
Anyone that claims the US has not done, and is not doing, greatly more than their fair share, regardless of whether it is about export quantity or price, is simply not living in the real world, or has an agenda to push.
Once again – the US is providing for ALL domestic demand, including ethanol, is providing for ALL export demand, and sill maintains an appx 1 to 2 billion bushel domestic reserve.
We also effectively maintain an appx 5 billion bushel further domestic reserve thru the ethanol industry, which the ethanol industry demonstrated in 2012. With low crop yields and substantial losses related to the drought, and production well below projected, the US ethanol industry reduced corn their usage in 2012 by almost the entire amount below projections.
Egypt has huge oil reserves and revenue. It is not our responsibility to provide this rich nation cheap, subsidized food. We already provide the majority of their corn import demand and have for many years. And we do so at fair, market prices. It is Egypt’s responsibility to provide for their people – and they have plenty of money to do so should the ruling elite so choose.

March 26, 2013 7:07 pm

One more for Richard Verney. From Rami Zurayk, professor of agricultural and food sciences at the American University of Beirut and author of Food, Farming and Freedom: Sowing the Arab Spring. He addresses pretty much exactly what I said:

Already, in Egypt and Yemen, more than 40% of the population live below the poverty line and suffer from some form of malnutrition. Most of the poor in these countries have no access to social safety nets.
“Bread riots” have been occurring regularly since the mid 1980s, following policies brought to us by the World Bank and the International Monetary Fund. Among these were the reduction of agricultural subsidies and the encouragement of production of fruits and vegetables for export, at the expense of investing in local grain production. Export of value-added produce and the import of basic commodities such as wheat were monopolised by a small group of “entrepreneurs” protected by the security state who financially backed the ruling elite…. The US gave Egypt around $1.7bn last year, exceeded only by the $2.4bn it gave to Israel.

Not only are we providing the majority of their corn (and other) imports, we are sending huge sums of other aid.
Again – the “State” and the ruling elite have vast sums of oil money at their control, yet seemingly refuse to use it.

March 26, 2013 8:18 pm

It’s not the CO2, it’s the sun’s energy infused in plants and fossil fuels for millions and millions of years from the very beginnings of photosynthesis and released when carbon is burned in the presence of oxygen. (Convection.) The newly formed molecules absorb the released heat setting those molecules in motion, expanding, rising, “warm air rises.” Remember that phrase from your old highschool physics classes on the weather? Pots of water heating on burners demonstrating the creation of currents in a fluid medium later to be applied to the creation of air currents, warm air rises. It takes a huge amount of energy to lift those masses into the atmosphere. There, that same energy drives the giant engines of weather in the atmosphere. Additional energy and water vapor arrive in the atmosphere following evaporation from surface waters. Nature and man both release CO2 but nature’s is put to good use. The weather delivers carbon and water to the forests and fields of the planet. Carbon for the growth of the plant and water for the nourishment of the plant. We consume those plants for our growth and development but it is the sun’s energy infused in the plant during photosynthesis that gives us our spark of life, the energy to do our work. Nature’s release of carbon and water travel circular paths. See the Keeling Curve for a picture of the earth “breathing”. The curve not only records nature’s output of CO2 but man’s as well! Nature uses the same amounts of CO2 and H2O, approximately, over and over and over again. Nature is first and foremost a recycler. Man on the other hand is not a recycler. When he runs out of fuel he simply digs and drills for more. Actually, his output has nowhere to go .. Forest and field have already absorbed their quota of CO2. Man’s CO2 accumulates. More matter means more energy. E = M , sort of. In any event, follow the energy. Global warming/climate change is the weather, EXTREME weather, weather on the rampage, gales gone wild, weather on steroids – HEAT energy. Follow the energy.

March 27, 2013 1:26 am

Gail Combs says:
March 26, 2013 at 4:42 pm
Gail,
We have been there several times too. In summary:
CO2 is well mixed in over 95% of the atmosphere. It is only highly variable in the first few hundred meters over land. CO2 levels measured in the bulk of the atmosphere don’t change with more than +/- 8 ppmv over a year (that is about 2% of the scale), the huge seasonal exchanges (+/- 20% of all CO2 in the atmosphere) included. Any scientist worth his/hers money calls that well mixed.
Well mixed doesn’t mean that any release or capturing of CO2 at any point on earth is mixed in instantly all over the earth. It only says that changes at any point are mixed in over a reasonable period of time. Which is the case for CO2.
Lucy Skywalker (whom I did meet a few years ago) still believes the late Jaworowski, whose knowledge ended in 1992, firmly refuted in 1996 by the work of Etheridge e.a. on three Law Dome ice cores. Anybody who denies that the average age of enclosed gas bubbles is younger than of the surrounding ice layer in ice cores, in my opinion has stopped learning about ice cores. And declaring that CO2 levels migrate from lower to higher levels is as good as closing the door for any credibility forever. Others like Glassmann are constantly misinterpreting what is said by others, so that any real discussion with him is impossible.
Then the Japanese satellite data: what is published unfortunately are flux data, not the absolute CO2 levels (although they must have them too). Of course there are huge fluxes within a year over the seasons, but that says next to nothing about the change in total CO2 levels.
Any global temperature change leads to changes in total CO2 of the atmosphere. Over the seasons some 5 ppmv for a change of 1Ā°C (mainly caused by vegetation in the NH). Over (very) long time spans that is 8 ppmv/Ā°C. That is all. We are now at 100 ppmv increase for some 1Ā°C increase since the LIA, at an incredible fixed ratio with human emissions over the past 110 years. If you know of any natural process that can deliver 92 ppmv CO2 in the atmosphere in lockstep with human emissions, I am very interested…

March 27, 2013 1:40 am

Francis X. Farley says:
March 26, 2013 at 8:18 pm
While I agree with the fate of human CO2 (as cause of the increase in total mass, not as the fate of individual molecules), the amount of energy released by burning fuels is really futile compared to what the sun sends to us. The amount of water and energy released is less than 0.01% of insolation and the natural water cycle, if I remember well from my calculations in the past. Hardly of influence on the global balance of both energy and water.
Even so, local releases in towns are local heat islands and cause upwind conditions wich may affect local -extreme- rainfall over towns. But there is no global trend at all in extreme weather, be it rain, drought, tornado’s, hurricanes,…

Gail Combs
March 27, 2013 5:40 am

Ferdinand Engelbeen, we will have to wait and see what happens as the earth descends into a cooling cycle. However after seeing what Hansen did to the temperature data and what Mauna Loa does with their CO2 data:with outliers and without outliers, I think we will have a mile of ice sitting on Chicago before these so called scientists actually give us any real data.
The lying and fabrication of ‘scientists’ is reaching the point where even the general public is noticing, thank goodness. I can not believe, based on all the other evidence available that you still defend the CO2 part of the scam. The fact that a prominent scientist like Dr. Jaworowski was FIRED rather than allowed to further investigate ice core measurement methodology used for CO2, tells me the ‘Consensus’ had something to hide and were not interested in actual science.

Gail Combs
March 27, 2013 6:07 am

Ferdinand Engelbeen says:
March 27, 2013 at 1:26 am
….. In summary:
CO2 is well mixed in over 95% of the atmosphere. It is only highly variable in the first few hundred meters over land….
>>>>>>>>>>>>>>>>>>>>>>>>
And there you are WRONG. You forgot about the volcanoes: Map “…Kilauea volcano in Hawaii has been erupting nearly continuously since 1983….” and Weekly Volcanic Activity Report

IG reported that during 13-17 March seismicity at Tungurahua was high. On 13 March ash plumes rose 1-3 km above the crater, and generated ashfall in Choglontus and Puela. The next day nearly continuous emissions of gas and ash rose 500 m. Explosions produced ash plumes that rose 3 km and blocks rolled 500 m down the flanks. On 15 March ash plumes drifted SE and W. An explosion generated an ash plume that rose 4 km and drifted E. A pyroclastic flow occurred near the crater.
On 16 March the eruptive activity at Etna changed from Strombolian explosions to lava fountaining, with the highest jets rising 600-800 m above the crater rim. Several lightning flashes within the eruptive cloud were observed….. http://www.volcano.si.edu/index.cfm

The Role of Explosive Volcanism During the Cool Maunder Minimum
Abstract
Understanding of the natural climate variability is crucial for evaluating the anthropogenic contribution to global warming. In particular, external forcing factors such as solar irradiation changes and aerosol forcing from explosive volcanism need to be captured accurately in order to detect and quantify the emerging signal. The short instrumental period limits our options to estimate the magnitude of external forcing through absence of the full range in magnitudes of the forcing factors as well as by lack of their low frequency representation. Thus, we are forced to use proxies to expand our record. Reconstructions of solar irradiance have often employed sunspot observations as a measure of solar activity. A striking feature has always been the Maunder Minimum, a multi-decadal period where the sunspots almost entirely disappeared. It is generally associated with reduced solar irradiance. Unusually cold conditions in Western Europe, especially during the late Maunder Minimum from 1675-1705, have often been used synonymous for the Little Ice Age. This link between the solar irradiance and temperatures during the Maunder Minimum has been applied for estimating either the magnitude of the low frequency solar irradiance changes while assuming a particular climate sensitivity, or conversely, to estimate the climate sensitivity assuming a magnitude of solar irradiance change. In doing so, other potential causes of the cool conditions were ignored. Interestingly, the climate conditions during the Maunder Minimum don’t remain cold over the entire period but exhibit a number of very cold, pulse-like episodes of a few years length. Here, the role of explosive volcanism superposed on solar irradiance changes during the late Maunder Minimum is evaluated. Using the fully coupled NCAR Climate System Model different ice core based volcanic forcing series are applied and combined with solar irradiance reconstructions. Not only temporal radiative balance impacts of the forcings are analyzed but also the spatially characteristical evolution of the signals. These fingerprints are then verified by a series of high resolution proxy reconstructions of European and Northern Hemisphere climate. Through this comparison of model with proxy data we quantify the volcanic cooling during this period and highlight the danger of estimating the climate sensitivity when omitting other factors.

Lester Via
March 27, 2013 7:52 am

A. Scott says:
March 26, 2013 at 6:27 pm
“lets take the 1971-1991 average of $2.28 a bushel and compare to the 2011 price of $5.18. That would be a 127% increase from 1971 to 2011, a 40 year period. That would be an average annual increase of just over 3%.”
————————————————————————————————————————–
When expressing an increase as an average annual percentage over a long time period, it generally means relative to the prior year rather than the beginning number. This allows the increase to be compared to the average rate of inflation over the same time period. When done in this manner the annual increase in the price of corn is 2.07% rather than “just over 3%”.

Lester Via
March 27, 2013 8:21 am

But then, using the average price of corn for the period of 1971-1991 and comparing it to the 2011 price and calling it a 40 year period doen’t seem kosher either. A 30 year period (1981 to 2011) woud be closer- in which case the average annual increase in price would be 2.77%, still relatively low

Lester Via
March 27, 2013 9:00 am

But then again – a little research at http://futures.tradingcharts.com/hist_CN.html shows corn prices at the end of 1971 to be $1.18 and $5.56 at the end of 2011- an average annual increase of 3.33%. But the price at the end of 2012 was $7.10 a price jump of 27.7% in one year.
It seems commodity prices can be used to make whatever point one wants to make simply by selecting dates.

March 27, 2013 11:34 am

It is amazing how people cling to ideas that are clearly wrong. In my lab, I use a 250W Weller heat gun that emits hot air at about 400C. Once stable, the metal nozzle will get up around that temperature. Can you visualize me holding this heat gun in one hand and a thermometer in the other? I want to use the heat gun to emulate “back radiation” and heat up the thermometer by 33C. I can’t use air flow convection to assist my heating, because that is an atmospheric cooling mechanism, not an atmospheric heating mechanism, so I will hold the thermometer level or closer to the floor than the heat gun. The metal nozzle radiates IR, of course it does, so that’s my source. How close does the thermometer have to be to the IR radiating metal nozzle to increase its temperature by 33C? You tell me: is it even physically possible? Can I do it without the nozzle and thermometer touching and enabling heating via conduction? If you think it’s physically possible, imagine “back radiating” CO2 all around you having the same effect. It would be readily apparent. In some cases, it would be lethal. Think, people, think.
Imagine introducing a CO2 molecule between the metal nozzle and the thermometer. Set it free. What can it do to increase the thermal coupling between the nozzle and thermometer? As soon as you set it loose, it will race toward the ceiling and carry thermal energy with it, so, clearly and inarguably, it’s an agent of cooling. Want to convince me otherwise? Easy, just show me your lab test results.

March 27, 2013 1:08 pm

Gail Combs says:
March 27, 2013 at 5:40 am
Dear Gail, the CO2 graphs you did send were not from Mauna Loa. The noisy one is from Neuglobsow ( http://www.igb-berlin.de/locations.html ), north of Berlin (Germany) midst a natural park, where CO2 levels go skyhigh at night if there is inversion and a lot lower during the growing season on sunny days… The second is from Mace Head, coastal (Ireland, http://macehead.org/index.php?option=com_content&view=article&id=46&Itemid=27 ). The difference between the first station and the second is exactly the difference between measuring in the 5% of the atmosphere where the sources and sinks are huge and the air masses are not mixed fast enough to level the differences, or measuring in 95% of the atmosphere where the mixing most of the time is adequate enough to level off the differences within a reasonable time frame.
The real, unaltered hour by hour data from Mauna Loa (MLO) and the South Pole (SPO) can be compared to the “cleaned” data, where the outliers are removed to make daily and monthly averages:
http://www.ferdinand-engelbeen.be/klimaat/klim_img/co2_mlo_spo_raw_select_2008.jpg
Note the difference in noise between MLO and SPO, but also compare the scale of both to the scales used in Mace Head and the Neuglobsow CO2 levels.
The variability at MLO is mostly between +4 ppmv (with downwind from the volcanic vents) and -4 ppmv (with upwind from vegetation in the valleys, mostly in the afternoon) around the seasonal and long term trend. That is all. These outliers from known local origin are not used for daily, monthly and yearly averages. But is doesn’t make any difference in the average or trend over a year. SPO has not such problems, because far away from volcanoes and no vegetation for thousands of km. But the harsh conditions make that more mechanical problems arise. The net result is that SPO shows exactly the same trend, but a lag of about 18n months with the CO2 levels of MLO. So where is the manipulation?
About volcanoes: CO2 emissions from land volcanoes are not of interest here, but the injection of enormous amounts of SO2 into the stratosphere is of interest: The Pinatubo, a one in over 100 years event, caused an extra cooling of maximum 0.6Ā°C over some three years, due to sulphate and other aerosols. The extra CO2 released had less effect than the cooling: the CO2 increase shows a dip, not an extra increase of CO2 in the atmosphere. That dip is within the natural variability of +/- 1 ppmv around the trend which is about 2 ppmv/yr nowadays.

March 27, 2013 2:05 pm

Ferdinand:
I am writing this post as a courtesy to show that I am not ignoring your post addressed to me at March 26, 2013 at 3:48 pm.
I stated my view as clearly as I could in my post at March 26, 2013 at 4:35 am which your post answers.
As you say in introduction to your view in your post

We have been there for several years now, but for new readers not familiar with our discussion, I will give my opinion again in short:

Indeed, we have been debating the matter for over a decade in several places and interested people can search the WUWT files to obtain complete knowledge of our different views.
The basis of our difference is your assertion of the ā€˜mass balance argumentā€™ and my rejection of that as being a circular argument. I see no purpose in repeating that again here when interested people can see our previous debates on WUWT.
As you know, Arthur Rorsch, Dick Thoenes and I published a paper which says the same as Salby later also said. You dispute our findings and Salbyā€™s later but very similar findings. I dispute your finding. And I am willing to allow others to assess your, our and Salbyā€™s analyses for themselves.
Richard

March 28, 2013 12:06 am

Lester Via – I used the USDA FG Yearbook, which contains data including price received at the farm per bushel. I used the 1971 to 1991 average to try to somewhat filter out volatility in the older number. As you note an approach such as your generally shows a smaller average annual increase than I showed.
In any event we both reach similar conclusions. Those that attack ethanol because is taking food from poor people by reducing export supply and driving up prices are simply not accurate. Whether the average annual increase is 2% or 3% is immaterial to those claims. The facts are corn prices on average have barely increased in 40 years. If we were to factor in inflation corns average prices if I recall have actually fallen.
By averaging the start and not doing so – using the speculation inflated price of 2011 – we again overstate the true increase. If we used say the 1971-1991 average vs the 2005-2011 average, which is inflated by a couple sets of spike years – we would be comparing $2.27 to $4.04 – a 77% total – or appx 1.9% average annual increase.
The current corn price you note is completely irrelevant to the discussion – it is based on the poor harvests last year due to the drought and the markets concerns over more of the same for this years crop. With investor speculation of another year of lower production and little change in demand – the price is being bid up as a result.
Has little to do with corn used for ethanol as I’ve shown above. In the case of a true shortage ethanol producers simply cut back production and use less corn – exactly as they did in 2012’s compromised crop year.

rgbatduke
March 28, 2013 9:50 am

Actually ā€œIntensityā€ or ā€œradiant intensityā€ as it applies to electro-magnetic radiation has SI units of Watts per steradian.
Never is it Watts per metre sqared.

You mean never as in “all the time”, as in nearly every textbook on electromagnetic radiation ever written? Including the ones I’ve written? You mean never as in not here:
http://webpages.ursinus.edu/lriley/courses/p212/lectures/node26.html
http://en.wikipedia.org/wiki/Poynting_vector
(units of watts/m^2, intensity commonly defined as the magnitude of the Poynting vector, SI units in common with sound intensity: http://en.wikipedia.org/wiki/Sound_intensity)? Never as in one of the most common definitions of received power through any given surface is that the power is the flux of Poynting vector.
You are quite correct that it is also often given as watts per steradian, basically the Poynting vector over r^2. You are completely and categorically incorrect when you state that average power or average intensity have no meaning. They are all that most devices for measuring radiation intensity — which do not have a clue about steradians or solid angles, and depend entirely on integrated flux of the Poynting vector through a detector surface — ever measure, as outside of the comparatively low frequency RF part of the spectrum we cannot measure real-time variations in \vec{S}.
Again, physics textbooks galore, all the way up to graduate level textbooks, contradict you. And what possible virtue is there in asserting that average power is meaningless quite aside from the fact that it is a false statement? To enable you to make an argument that every camera, every optical transducer in the universe contradicts? Even your eye responds to average power, not instantaneous power, because it takes a certain average number of photons to trigger rhodopsins in retinal cells. Every digital camera. Photographic film. Photocells. You tell me what optical frequency radiation devices can respond to electromagnetic fields varying on a timescale of 10^{-15} seconds.
rgb

April 1, 2013 9:18 am

Not sure of the importance or relevance to the debate in the comments. I have checked the math and the first section checks out. However in doing the research to validate the post (I plan to replicate it on my blog) I found the IPCC’s listed value for forcing at 3.7 not 2.7. The original calculated value is 4 however they adjusted it as a fudge factor to 3.7. This would alter the original posted value of .8/140 years to roughly 1.01/140 years. an almost arbitrary difference. It may also explain Stan’s confusion over discrepancies between his original calculation.
As for the second part the math looks fine past the logarithmic calculation. Im a bit iffy on how the log was used to get the fraction of the difference. This doesn’t invalidate the approach but If someone could post a quick explanation of why it was used in this way (log(310/280)/log(2)) it would be much appreciated.